INSIGHTSIAS SIMPLYFYING IAS EXAM PREPARATION

INSTA 75 Days REVISION PLAN

UPSC Prelims 2020

INSTA Tests 65 to 68 (GS)

KEY & EXPLANATIONS

www.insightsonindia.com prelims.insightsonindia.com | mains.insightsonindia.com Telegram: insightsIAStips | FB: insightsonindia | TW: vinaygb | YT: INSIGHTS IAS BENGALURU | DELHI | HYDERABAD

Copyright © by Insights IAS All rights are reserved. No part of this document may be reproduced, stored in a retrieval system or transmitted in any form or by any means, electronic, mechanical, photocopying, recording or otherwise, without prior permission of Insights IAS.

Insta 75 Days Revision Plan for UPSC Civil Services Prelims – 2020

This document is the compilation of 100 questions that are part of InsightsIAS famous INSTA REVISION initiative for UPSC civil services Preliminary examination – 2020 (which has become most anticipated annual affair by lakhs of IAS aspirants across the country). These questions are carefully framed so as to give aspirants tough challenge to test their knowledge and at the same time improve skills such as intelligent guessing, elimination, reasoning, deduction etc – which are much needed to sail through tough Civil Services Preliminary Examination conducted by UPSC.

These questions are based on this INSTA Revision Plan which is posted on our website (www.insightsonindia.com). Every year thousands of candidates follow our revision timetable – which is made for SERIOUS aspirants who would like to intensively revise everything that’s important before the exam.

Those who would like to take up more tests for even better preparation, can enroll to Insights IAS Prelims Mock Test Series – 2020 (https://prelims.insightsonindia.com). Every year toppers solve our tests and sail through UPSC civil services exam. Your support through purchase of our tests will help us provide FREE content on our website seamlessly.

Wish you all the best!

Team InsightsIAS

INSTA 75 Days REVISION PLAN for Prelims 2020 - InstaTests

DAY – 65 (InstaTest-65)

1. Consider the following statements regarding International Treaty on Plant Genetic Resources for Food and Agriculture 1. It is also known as the International Seed Treaty. 2. The Treaty helps maximize the use and breeding of all crops and promotes development and maintenance of diverse farming systems. 3. This treaty recognizes and protects the farmer rights. Which of the statements given above is/are correct? (a) 1 and 2 only (b) 1 and 2 only (c) 3 only (d) 1, 2 and 3

Solution: D

• International Treaty on Plant Genetic Resources for Food and Agriculture is popularly known as the International Seed Treaty, is a comprehensive international agreement in harmony with the Convention on Biological Diversity, which aims at guaranteeing food security through the conservation, exchange and sustainable use of the world’s plant genetic resources for food and agriculture (PGRFA), as well as the fair and equitable benefit sharing arising from its use. It also recognizes farmers’ rights, subject to national laws to:

• the protection of traditional knowledge relevant to plant genetic resources for food and agriculture; • the right to equitably participate in sharing benefits • the right to participate in making decisions etc. The Treaty helps maximize the use and breeding of all crops and promotes development and maintenance of diverse farming systems.

2. Consider the following statements regarding Repco Bank 1. Repco Bank is a multi-state cooperative society established for rehabilitation of repatriates from and Sri Lanka. 2. It is controlled by Ministry of Finance.

www.insightsonindia.com 1 Insights IAS INSTA 75 Days REVISION PLAN for Prelims 2020 - InstaTests

3. It is operated only in the South Indian states of Andhra Pradesh, Karnataka, Kerala and Tamil Nadu. Which of the statements given above is/are correct? (a) 1 only (b) 2 and 3 only (c) 1 and 3 only (d) 1, 2 and 3

Solution: C

Repco Bank

• Union Home Minister received a dividend cheque of Rs 15.26 crore from the representatives of the Repco Bank, a multi-state cooperative finance and development bank controlled by the Home Ministry. • The Repco Bank is a multi-state cooperative society established in 1969 by the central government for rehabilitation of repatriates from Myanmar and Sri Lanka. • It is operated only in the South Indian states of Andhra Pradesh, Karnataka, Kerala and Tamil Nadu. • As on March 31, 2019 the government of held 49.15 per cent of the share capital, four southern state governments held 6.24 per cent and the remaining 45 per cent was held by individual repatriates. • Repatriates Cooperative Finance and Development Bank Ltd., (REPCO BANK) was registered on 19.11.1969 as a Cooperative Society under the relevant provisions of Madras Cooperative Societies Act, 1961 with Jurisdiction over the State of Tamil Nadu, Andhra Pradesh, Karnataka, Kerala and Union Territory of Puducherry for the purpose of promoting the rehabilitation activities for repatriates from neighboring countries mainly from Sri Lanka and Burma. Though originally registered under the Madras Cooperative Societies Act, 1961, the Bank is deemed to be registered under the Multi State Cooperative Societies Act, 2002. • The share capital of the Bank is controlled by Government of India, Governments of 4 Southern States namely Tamil Nadu, Kerala, Karnataka, Andhra Pradesh and Repatriates.

www.insightsonindia.com 2 Insights IAS INSTA 75 Days REVISION PLAN for Prelims 2020 - InstaTests

3. Consider the following statements regarding Western Dedicated Freight Corridor (DFC) 1. It begins at Dadri in Uttar Pradesh and till the Jawaharlal Nehru Port Trust, near Mumbai. 2. The Dedicated Freight Corridor Corporation of India (DFCCIL) has been established to carry out the project. 3. The project will be funded by loan from World Bank. Which of the statements given above is/are correct? (a) 1 and 2 only (b) 2 and 3 only (c) 1 only (d) 1, 2 and 3

Solution: A

Western Dedicated Freight Corridor (DFC)

• It is a broad-gauge corridor. • The 1,504-km western freight corridor begins at Dadri in Uttar Pradesh and stretches till the country’s largest container port — Jawaharlal Nehru Port Trust, near Mumbai. • In October 2006, a dedicated body, the Dedicated Freight Corridor Corporation of India (DFCCIL) has been established to carry out the project. • The project will be funded by a soft loan of $4bn provided by Japan International Cooperation Agency under special terms for economic partnership (STEP). Important places in this route:

• It passes through Vadodara, Ahmadabad, Palanpur, Phulera and Rewari. • Passing through U.P., Haryana, Rajasthan, Gujarat and Maharashtra. The project will eventually be linked to the Eastern DFC to form four hubs known as India’s Golden Quadrilateral including Delhi, Mumbai, Chennai and Kolkata. About Dedicated Freight Corridor Corporation of India (DFCCIL): The DFCCIL is a corporation run by the Ministry of Railways (India) to undertake planning & development, mobilization of financial resources and construction, maintenance and operation of the Dedicated Freight Corridors. DFCC has been registered as a company under the Companies Act 1956 on 30 October 2006.

www.insightsonindia.com 3 Insights IAS INSTA 75 Days REVISION PLAN for Prelims 2020 - InstaTests

4. Consider the following statements regarding Jal Jeevan Mission 1. It aims to create local infrastructure for rainwater harvesting, groundwater recharge and management of household waste water for reuse in agriculture. 2. The chief objective of the mission is to provide piped water supply to all rural and urban households by 2024. 3. India has 20% of freshwater resources of the entire world. Which of the statements given above is/are correct? (a) 1 and 3 only (b) 2 only (c) 1 and 2 only (d) 1, 2 and 3

Solution: C

• Jal Jeevan Mission was announced in August 2019. • The chief objective of the Mission is to provide piped water supply (Har Ghar Jal) to all rural and urban households by 2024.It also aims to create local infrastructure for rainwater harvesting, groundwater recharge and management of household waste water for reuse in agriculture • India has 16% of the world population, but only 4% of freshwater resources. Depleting groundwater level, overexploitation and deteriorating water quality, climate change, etc. are major challenges to provide potable drinking water • Jal Jeevan Mission will focus on integrated demand and supply management of water at the local level.

5. Consider the following statements regarding STRIPS – Separate Trading of Registered Interest and Principal of Securities 1. STRIPS are the securities created by way of separating the cash flows associated with a regular G-Sec and the final principal payment to be received from the issuer, into separate securities. 2. They are essentially Zero-Coupon Bonds (ZCBs). 3. They are created out of existing securities only and like other securities, they are issued through auctions. Which of the statements given above is/are correct? (a) 1 and 2 only (b) 2 and 3 only

www.insightsonindia.com 4 Insights IAS INSTA 75 Days REVISION PLAN for Prelims 2020 - InstaTests

(c) 1 only (d) 1, 2 and 3

Solution: A

STRIPS – Separate Trading of Registered Interest and Principal of Securities. – STRIPS are the securities created by way of separating the cash flows associated with a regular G-Sec i.e. each semi-annual coupon payment and the final principal payment to be received from the issuer, into separate securities. They are essentially Zero-Coupon Bonds (ZCBs). However, they are created out of existing securities only and unlike other securities, are not issued through auctions. Stripped securities represent future cash flows (periodic interest and principal repayment) of an underlying coupon bearing bond. Being G-Secs, STRIPS are eligible for SLR. All fixed coupon securities issued by Government of India, irrespective of the year of maturity, are eligible for Stripping/Reconstitution, provided that the securities are reckoned as eligible investment for the purpose of Statutory Liquidity Ratio (SLR) and the securities are transferable. The detailed guidelines of stripping/reconstitution of government securities is available in RBI notification IDMD.

For example, when ₹100 of the 8.60% GS 2028 is stripped, each cash flow of coupon (₹ 4.30 each half year) will become a coupon STRIP and the principal payment (₹100 at maturity) will become a principal STRIP. These cash flows are traded separately as independent securities in the secondary market. STRIPS in G-Secs ensure availability of sovereign zero coupon bonds, which facilitate the development of a market determined zero coupon yield curve (ZCYC). STRIPS also provide institutional investors with an additional instrument for their asset liability management (ALM). Further, as STRIPS have zero reinvestment risk, being zero coupon bonds, they can be attractive to retail/non-institutional investors. Market participants, having an SGL account with RBI can place requests directly in e-kuber for stripping/reconstitution of eligible securities (not special securities). Requests for stripping/reconstitution by Gilt Account Holders (GAH) shall be placed with the respective Custodian maintaining the CSGL account, who in turn, will place the requests on behalf of its constituents in e-kuber.

6. Consider the following statements regarding Complaint Management System (CMS) 1. It is a software application to facilitate RBI’s grievance redressal process. 2. Customers can lodge complaints only against commercial banks. 3. The complaint would be directed to the appropriate office of the Ombudsman. Which of the statements given above is/are correct? (a) 1 and 2 only

www.insightsonindia.com 5 Insights IAS INSTA 75 Days REVISION PLAN for Prelims 2020 - InstaTests

(b) 2 and 3 only (c) 1 and 3 only (d) 1, 2 and 3

Solution: C

Complaint Management System (CMS) Launched by the Reserve Bank of India (RBI), the Complaint Management System (CMS) is a software application to facilitate RBI’s grievance redressal process.

• Aim: to improve customer experience in timely redressal of grievances. How it works?

• Customers can lodge complaints against any regulated entity with public interface such as commercial banks, urban cooperative banks, Non-Banking Financial Companies (NBFCs). The complaint would be directed to the appropriate office of the Ombudsman/Regional Office of the RBI. Benefits:

• The application improves transparency by keeping the complainants informed through auto-generated acknowledgements and enabling them to track the status of their complaints and file appeals online against the decisions of the Ombudsmen, where applicable. • Complainants can also voluntarily share feedback on their experience in obtaining redressal.

7. Consider the following statements regarding Lymphatic filariasis 1. It is caused by infection with parasitic worms living in the lymphatic system. 2. It cause abnormal enlargement of body parts, and leading to severe disability and social stigmatization of those affected. Which of the statements given above is/are correct? (a) 1 only (b) 2 only (c) Both 1 and 2 (d) Neither 1 nor 2

www.insightsonindia.com 6 Insights IAS INSTA 75 Days REVISION PLAN for Prelims 2020 - InstaTests

Solution: C

• Also called as elephantiasis, it is caused by infection with parasitic worms living in the lymphatic system. • The larval stages of the parasite (microfilaria) circulate in the blood and are transmitted from person to person by mosquitoes. • It may Cause abnormal enlargement of body parts, and leading to severe disability and social stigmatization of those affected. • Lymphatic filariasis poses a grave threat to India. Over 40% of worldwide cases are found in India.

8. Consider the following statements regarding National Investment and Infrastructure Fund (NIIF) 1. It an investment vehicle for funding commercially viable green field, brown field and stalled infrastructure projects. 2. The Indian government is investing 51% and the rest of the corpus is to be raised from third-party investors. Which of the statements given above is/are correct? (a) 1 only (b) 2 only (c) Both 1 and 2 (d) Neither 1 nor 2

Solution: A

National Investment and Infrastructure Fund (NIIF)

• Canada’s largest pension fund Canada Pension Plan Investment Board (CPPIB) has agreed to invest about $600 million in National Investment and Infrastructure Fund (NIIF) through the NIIF Master Fund. About NIIF:

• The government had set up the ₹40,000 crore NIIF in 2015 as an investment vehicle for funding commercially viable green field, brown field and stalled infrastructure projects.

www.insightsonindia.com 7 Insights IAS INSTA 75 Days REVISION PLAN for Prelims 2020 - InstaTests

• The Indian government is investing 49% and the rest of the corpus is to be raised from third-party investors such as sovereign wealth funds, insurance and pension funds, endowments, etc. • NIIF’s mandate includes investing in areas such as energy, transportation, housing, water, waste management and other infrastructure-related sectors in India. • NIIF currently manages three funds each with its distinctive investment mandate. The funds are registered as Alternative Investment Fund (AIF) with the Securities and Exchange Board of India (SEBI). The three funds are:

• Master Fund: Is an infrastructure fund with the objective of primarily investing in operating assets in the core infrastructure sectors such as roads, ports, airports, power etc. • Fund of Funds: Managed by fund managers who have good track records in infrastructure and associated sectors in India. Some of the sectors of focus include Green Infrastructure, Mid-Income & Affordable Housing, Infrastructure services and allied sectors. • Strategic Investment Fund: Is registered as an Alternative Investment Fund II under SEBI in India. The objective is to invest largely in equity and equity-linked instruments. It will focus on green field and brown field investments in the core infrastructure sectors.

9. Consider the following statements regarding Laffer Curve 1. The Laffer Curve describes the relationship between tax rates and total tax revenue. 2. If taxes are too high along the Laffer Curve, then they will increase total tax revenue. Which of the statement above is/are correct? (a) 1 only (b) 2 only (c) Both 1 and 2 (d) Neither 1 nor 2

Solution: A

• The Laffer Curve describes the relationship between tax rates and total tax revenue, with an optimal tax rate that maximizes total government tax revenue.

www.insightsonindia.com 8 Insights IAS INSTA 75 Days REVISION PLAN for Prelims 2020 - InstaTests

• If taxes are too high along the Laffer Curve, then they will discourage the taxed activities, such as work and investment, enough to actually reduce total tax revenue. In this case, cutting tax rates will both stimulate economic incentives and increase tax revenue. • The Laffer Curve was used as a basis for tax cuts in the 1980’s with apparent success, but criticized on practical grounds on the basis of its simplistic assumptions, and on economic grounds that increasing government revenue might not always be optimal. • The Laffer Curve is based on the economic idea that people will adjust their behavior in the face of the incentives created by income tax rates. Higher income tax rates decrease the incentive to work and invest compared lower rates. If this effect is large enough, it means that at some tax rate, and further increase in the rate will actually lead to decrease in total tax revenue. For every type of tax, there is a threshold rate above which the incentive to produce more diminishes, thereby reducing the amount of revenue the government receives.

10. Consider the following statements regarding Electoral bond scheme 1. An electoral bond is like a promissory note that can be bought by any Indian citizen or company incorporated in India from select branches of State Bank of India. 2. The bonds will be issued in multiples of Rs 1,000, Rs 10,000, Rs 100,000 and Rs 1 crore (the range of a bond is between Rs 1,000 to Rs 1 crore). 3. The electoral bond will be valid only for thirty days. Which of the statements given above is/are correct? (a) 1 and 2 only (b) 1 and 3 only (c) 3 only (d) 1, 2 and 3

Solution: A

• An electoral bond is like a promissory note that can be bought by any Indian citizen or company incorporated in India from select branches of State Bank of India. The citizen or corporate can then donate the same to any eligible political party of his/her choice. The bonds are similar to bank notes that are payable to the bearer on demand and are free of interest. An individual or party will be allowed to purchase these bonds digitally or through cheque. • Using electoral bonds is quite simple. The bonds will be issued in multiples of Rs 1,000, Rs 10,000, Rs 100,000 and Rs 1 crore (the range of a bond is between Rs 1,000 to Rs 1 www.insightsonindia.com 9 Insights IAS INSTA 75 Days REVISION PLAN for Prelims 2020 - InstaTests

crore). These will be available at some branches of SBI. A donor with a KYC-compliant account can purchase the bonds and can then donate them to the party or individual of their choice. Now, the receiver can encash the bonds through the party’s verified account. • The electoral bond will be valid only for fifteen days. https://www.business-standard.com/about/what-is-electoral-bond

11. Consider the following statements regarding Cash Reserve Ratio 1. It refers to the fraction of the total Net Demand and Time Liabilities (NDTL) of a Bank, that it has to maintain as cash deposit with the Reserve Bank of India (RBI). 2. The banks are not paid any interest on behalf of the RBI for parking the required cash. 3. Regional rural banks and co-operative banks are outside the purview of CRR. Which of the statements given above is/are correct? (a) 1 and 2 only (b) 2 and 3 only (c) 1 and 3 only (d) 1, 2 and 3

Solution: A

• Cash Reserve Ratio refers to the fraction of the total Net Demand and Time Liabilities (NDTL) of a Scheduled Commercial Bank held in India, that it has to maintain as cash deposit with the Reserve Bank of India (RBI). The requirement applies uniformly to all banks in the country irrespective of an individual bank’s financial situation or size. In contrast, certain countries e.g. China stipulates separate reserve requirements for ‘large’ and ‘small’ banks. • As per the RBI Act 1934, all Scheduled Commercial Banks (that includes public and private sector banks, foreign banks, regional rural banks and co-operative banks) are required to maintain a cash balance on average with the RBI on a fortnightly basis to cater to the CRR requirement. Non-Bank Financial Corporations (NBFCs) are outside the purview of this reserve requirement. Act also authorizes RBI to stipulate an additional or incremental CRR, which, however, has not been put in place by RBI. • Banks have to maintain 100 percent CRR on an average basis during the fortnight. That is, it is not necessary that on all days CRR has to be at 100%. With effect from December 28, 2002 all banks were required to maintain a minimum of 70 per cent of the required average daily CRR on all days of the fortnight. Later, with effect from the

www.insightsonindia.com 10 Insights IAS INSTA 75 Days REVISION PLAN for Prelims 2020 - InstaTests

fortnight beginning September 21, 2013 entities were required to maintain minimum CRR balances up to 95 per cent of the average daily required reserves for a reporting fortnight on all days of the fortnight. This was later reduced to 90 per cent with effect from the fortnight beginning April 16, 2016. • Traditionally, the amount held to cater to the CRR requirement was stipulated to be no lower than 3 percent and no higher than 20 percent of the total NDTL held in India. However, the RBI (amendment) Act, 2006 provides for removal of the floor and ceiling with respect to setting the CRR and authorizes the RBI to set the ratio in keeping with the broad objective of maintaining monetary stability in the economy. • Presently, banks are not paid any interest on behalf of the RBI for parking the required cash. If a bank fails to meet its required reserve requirements, the RBI is empowered to impose a penalty by charging a penal interest rate.

12. Consider the following statements regarding Fixed Income Money Market and Derivatives Association of India 1. It is an association of Scheduled Commercial Banks, Public Financial Institutions, Primary Dealers and Insurance Companies. 2. FIMMDA is a voluntary market body for the bond, money and derivatives markets. Which of the statements given above is/are correct? (a) 1 only (b) 2 only (c) Both 1 and 2 (d) Neither 1 nor 2

Solution: C

• The Fixed Income Money Market and Derivatives Association of India (FIMMDA), an association of Scheduled Commercial Banks, Public Financial Institutions, Primary Dealers and Insurance Companies was incorporated as a Company under section 25 of the Companies Act,1956 on June 3, 1998. FIMMDA is a voluntary market body for the bond, money and derivatives markets. FIMMDA has members representing all major institutional segments of the market. The membership includes Nationalized Banks such as State Bank of India, its associate banks and other nationalized banks; Private sector banks such as ICICI Bank, HDFC Bank; Foreign Banks such as Bank of America, Citibank, Financial institutions such as IDFC, EXIM Bank, NABARD, Insurance Companies like Life Insurance Corporation of India (LIC), ICICI Prudential Life Insurance Company, Birla Sun Life Insurance Company and all Primary Dealers.

www.insightsonindia.com 11 Insights IAS INSTA 75 Days REVISION PLAN for Prelims 2020 - InstaTests

• FIMMDA represents market participants and aids the development of the bond, money and derivatives markets. It acts as an interface with the regulators on various issues that impact the functioning of these markets. FIMMDA also plays a constructive role in the evolution of best market practices by its members so that the market as a whole operates transparently as well as efficiently.

13. Glyphosate, often seen in the news, is related to (a) Multidrug Resistance Tuberculosis (b) Bio-chemical agent to remove oil spills (c) Weedicide (d) None of the above

Solution: C

• Glyphosate’s increase resulted from the rapid adoption of genetically engineered crops and no-till farming practices, both of which incorporate glyphosate for weed control (Gianessi and Reigner, 2006). • The first triazine herbicide, atrazine, was discovered by J.R. Geigy, Ltd., in Switzerland (LeBaron et al., 2008). It was first registered in the United States in 1958. Atrazine has a range of trade names such as Marksman, Coyote, Atrazina, Atrazol, and Vectal (PAN, 2002) and is estimated to be the second most heavily used herbicide in the United States at 35,000 metric tons (77 million lbs) (Gianessi and Reigner, 2006). https://www.sciencedirect.com/topics/earth-and-planetary-sciences/glyphosate

14. Consider the following statements regarding Wholesale Price Index (WPI) 1. Primary Articles forms the major component of WPI. 2. It is measured on year-on-year basis i.e., rate of change in price level in a given month vis a vis corresponding month of last year. Which of the statements given above is/are correct? (a) 1 only (b) 2 only (c) Both 1 and 2 (d) Neither 1 nor 2

www.insightsonindia.com 12 Insights IAS INSTA 75 Days REVISION PLAN for Prelims 2020 - InstaTests

Solution: B

In general, reflects the rate of change in prices of all goods and services in an economy over a period of time. Every country has its own set of commodity basket to track inflation. While some countries use Wholesale Price Index (WPI) as their official measure of inflation and some others use the Consumer Price Index (CPI). The International Monetary Fund (IMF) statistics reveals that, while 24 countries use WPI as the official measure to track inflation, 157 countries use CPI. Conceptually these two measures of inflation stress different stages of price realization as well as composition: while WPI measures the change in price level at wholesale market, CPI measures the change in price level at retail level. In India, headline inflation is measured through the WPI (the latest base year 2011-12) – which consists of 697 commodities (services are not included in WPI in India). It is measured on year- on-year basis i.e., rate of change in price level in a given month vis a vis corresponding month of last year. This is also known as point to point inflation. Apart from WPI, CPI is also computed to capture inflation in India. In particular, four categories of CPI are computed – for Industrial Workers (CPI-IW), Urban Non-Manual Employees (CPI-UNME), Agricultural Labourers (CPI-AL) and Rural Labourers (CPI-RL). However, WPI is considered as the preferred measure of headline inflation due to its wider coverage. To overcome this lacuna, the Central Statistical Organization (on 18th February 2011) has introduced a new series of CPI (with 2010=100 as the base year), which would be calculated for all-India as well as States/UTs – with separate categorization for rural, urban and combined (rural + urban). Who publishes WPI in India and what does it show?

• Analysts use the numbers to track the supply and demand dynamics in industry, manufacturing and construction. The numbers are released by the Economic Advisor in the Ministry of Commerce and Industry. An upward surge in the WPI print indicates inflationary pressure in the economy and vice versa. The quantum of rise in the WPI month-after-month is used to measure the level of wholesale inflation in the economy. What is the difference between WPI and CPI inflation?

• While WPI keeps track of the wholesale price of goods, the CPI measures the average price that households pay for a basket of different goods and services. Even as the WPI is used as a key measure of inflation in some economies, the RBI no longer uses it for policy purposes, including setting repo rates. The central bank currently uses CPI or retail inflation as a key measure of inflation to set the monetary and credit policy. New series of WPI

• With an aim to align the index with the base year of other important economic indicators such as GDP and IIP, the base year was updated to 2011-12 from 2004-05 for the new series of Wholesale Price Index (WPI), effective from April 2017. www.insightsonindia.com 13 Insights IAS INSTA 75 Days REVISION PLAN for Prelims 2020 - InstaTests

How do you calculate Wholesale Price Index?

• The monthly WPI number shows the average price changes of goods usually expressed in ratios or percentages. • The index is based on the wholesale prices of a few relevant commodities available. • The commodities are chosen based on their significance in the region. These represent different strata of the economy and are expected to provide a comprehensive WPI value. • The advanced base year 2011-12 adopted recently uses 697 items.

15. Consider the following statements regarding India Brand Equity Foundation (IBEF) 1. It is a Trust established by the Confederation of Indian Industry (CII). 2. Its primary objective is to promote and create international awareness of the Made in India label. Which of the statement above is/are correct? (a) 1 only (b) 2 only (c) Both 1 and 2 (d) Neither 1 nor 2

Solution: B

India Brand Equity Foundation (IBEF)

• India Brand Equity Foundation (IBEF) is a Trust established by the Department of Commerce, Ministry of Commerce and Industry, Government of India. IBEF’s primary objective is to promote and create international awareness of the Made in India label www.insightsonindia.com 14 Insights IAS INSTA 75 Days REVISION PLAN for Prelims 2020 - InstaTests

in markets overseas and to facilitate dissemination of knowledge of Indian products and services. Towards this objective, IBEF works closely with stakeholders across government and industry • India, today, is well established as a credible business partner, preferred investment destination, rapidly growing market, provider of quality services and manufactured products; and, stands on the threshold years of unprecedented growth. • India’s Talent, Markets, Growth and Opportunity drive Brand India. • http://www.ibef.org is a knowledge centre for global investors, international policy- makers and world media seeking updated, accurate and comprehensive information on the Indian economy, states and sectors. IBEF regularly tracks government announcements in policy, foreign investment, macroeconomic indicators and business trends. IBEF works with a network of stakeholders – domestic and international – to promote Brand India.

16. Consider the following pairs of Martial arts and the states they are mainly associated with: Martial Arts States 1. : Odisha 2. : Maharashtra 3. : Arunachal Pradesh Which of the pairs given above is/are correctly matched? (a) 1 and 3 only (b) 2 and 3 only (c) 1 only (d) None

Solution: D

• Silambam is a weapon-based Indian martial art originating in modern-day Tamil Nadu in the Indian subcontinent and is estimated to have originated in approximately 1000 BCE. This ancient fighting style is mentioned in Tamil Sangam literature 400 BCE. • Gatka is the name of an Indian martial art associated with the Sikhs of the Punjab and the Tanoli and Gujjar communities of the mountainous regions of northern Pakistan who practice an early variant of the martial art. It is a style of stick-fighting, with wooden sticks intended to simulate swords. • Huyen langlon is an Indian martial art from . In the Meitei language, huyen means war while langlon or langlong can mean net, knowledge or art. Huyen langlon www.insightsonindia.com 15 Insights IAS INSTA 75 Days REVISION PLAN for Prelims 2020 - InstaTests

consists of two main components: thang-ta and sarit sarak. The primary weapons of huyen langlon are the thang and ta.

17. Consider the following statements regarding Effective Revenue deficit 1. The term Effective Revenue deficit was introduced in the Union Budget 2011-12. 2. Effective Revenue Deficit signifies that amount of capital receipts that are being used for actual capital expenditure of the Government. 3. Since its inception, Year on year the effective revenue deficit is decreasing. Which of the statements given above is/are correct? (a) 1 and 2 only (b) 2 only (c) 1 and 3 only (d) 1, 2 and 3

Solution: A

• Effective Revenue deficit is a new term introduced in the Union Budget 2011-12. While revenue deficit is the difference between revenue receipts and revenue expenditure, the present accounting system includes all grants from the Union Government to the state governments/Union territories/other bodies as revenue expenditure, even if they are used to create assets. Such assets created by the sub- national governments/bodies are owned by them and not by the Union Government. Nevertheless they do result in the creation of durable assets. • According to the Finance Ministry, such revenue expenditures contribute to the growth in the economy and therefore, should not be treated as unproductive in nature. In the Union Budget (2011-12) a new methodology has been introduced to capture the ‘effective revenue deficit’, which excludes those revenue expenditures (or transfers) in the form of grants for creation of capital assets. If this methodology is taken into account, the effective revenue deficit (revised estimates) for 2010-11 is only 2.3 per cent as against the revenue deficit of 3.4 per cent of GDP. The effective revenue deficit for 2011-12 is projected at 1.8 per cent as against the revenue deficit estimates of 3.4 per cent. • It may be noted that even though some grants may be allocated towards the creation of assets, financial allocation does not always result in physical outcomes. • Grants for creation of capital assets, as a concept, was introduced in the FRBM Act through the amendment in 2012. The Act defines grants for creation of capital assets as grants-in-aid given by the Central Government to state governments, autonomous

www.insightsonindia.com 16 Insights IAS INSTA 75 Days REVISION PLAN for Prelims 2020 - InstaTests

bodies, local bodies and other scheme implementing agencies for creation of capital assets which are owned by these entities. • In short, Effective Revenue Deficit is the difference between revenue deficit and grants for creation of capital assets. Effective Revenue Deficit signifies that amount of capital receipts that are being used for actual consumption expenditure of the Government.

18. Consider the following statements regarding National Strategy for Financial Inclusion 1. It was prepared by Ministry of Finance. 2. Every village to have access to a formal financial service provider within a reasonable distance of 15 KM radius. Which of the statements given above is/are correct? (a) 1 only (b) 2 only (c) Both 1 and 2 (d) Neither 1 nor 2

Solution: D

The National Strategy for Financial Inclusion 2019-2024 sets forth the vision and key objectives of the financial inclusion policies in India to help expand and sustain the financial inclusion process at the national level through a broad convergence of action involving all the stakeholders in the financial sector. The strategy aims to provide access to formal financial services in an affordable manner, broadening & deepening financial inclusion and promoting financial literacy & consumer protection. The National Strategy for Financial Inclusion for India 2019-2024 has been prepared by RBI under the aegis of the Financial Inclusion Advisory Committee and is based on the inputs and suggestions from Government of India, other Financial Sector Regulators viz., Securities Exchange Board of India (SEBI), Insurance Regulatory and Development Authority of India (IRDAI) and Pension Fund Regulatory and Development Authority of India (PFRDA). To achieve the vision of ensuring access to an array of basic formal financial services, a set of guiding objectives have been formulated with special relevance in the Indian context.

• Universal Access to Financial Services: Every village to have access to a formal financial service provider within a reasonable distance of 5 KM radius. The customers may be on boarded through an easy and hassle-free digital process and processes should be geared towards a less-paper ecosystem.

www.insightsonindia.com 17 Insights IAS INSTA 75 Days REVISION PLAN for Prelims 2020 - InstaTests

• Providing Basic Bouquet of Financial Services: Every adult who is willing and eligible needs to be provided with a basic bouquet of financial services that include a Basic Savings Bank Deposit Account, credit, a micro life and non-life insurance product, a pension product and a suitable investment product. • Access to Livelihood and Skill Development: The new entrant to the financial system, if eligible and willing to undergo any livelihood/ skill development programme, may be given the relevant information about the ongoing Government livelihood programmes thus helping them to augment their skills and engage in meaningful economic activity and improve income generation. • Customer Protection and Grievance Redressal: Customers shall be made aware of the recourses available for resolution of their grievances. About storing and sharing of customer’s biometric and demographic data, adequate safeguards need to be ensured to protect the customer’s Right to Privacy. • Effective Co-ordination: There needs to be a focused and continuous coordination between the key stakeholders viz. Government, the Regulators, financial service providers, Telecom Service Regulators, Skills Training institutes etc. to make sure that the customers are able to use the services in a sustained manner. The focus shall be to consolidate gains from previous efforts through focus on improvement of quality of service of last mile delivery viz., capacity building of Business Correspondents, creating payments system ecosystems at village levels to deepen the culture of digital finance leading to ease of use and delivery.

19. Idu-Mishmi tribe often seen in the news, is found in (a) Assam (b) Kashmir (c) Arunachal Pradesh (d) Manipur

Solution: C

• Idu-Mishmi tribe is the lone inhabitant tribe of Dibang Valley district. They are of mongoloid race and have distinctive dialect of Tibeto-Burma language. Idu-Mishmi tribe can be distinctively identified among other tribal groups of Arunachal Pradesh by their typical hairstyle, distinctive customs and artistic pattern embedded on their clothes. This tribe still maintains deep rooted aesthetic values in their day to day life with great pride and honour.

www.insightsonindia.com 18 Insights IAS INSTA 75 Days REVISION PLAN for Prelims 2020 - InstaTests

20. Consider the following statements regarding National Credit Guarantee Trustee Company Ltd [NCGTC] 1. It was set up by the Department of Financial Services, Ministry of Finance with a paid-up capital of ₹100 crore. 2. It acts as a common trustee company to manage and operate various credit guarantee trust funds. Which of the statement above is/are correct? (a) 1 only (b) 2 only (c) Both 1 and 2 (d) Neither 1 nor 2

Solution: B

• Subsequent to the Central Budget announcements to set up various credit guarantee funds, a common trustee company in the name and style of National Credit Guarantee Trustee Company Ltd [NCGTC] was set up by the Department of Financial Services, Ministry of Finance, Government of India to, inter alia, act as a common trustee company to manage and operate various credit guarantee trust funds. • NCGTC was incorporated under the Indian Companies Act, 1956 on March 28, 2014 with a paid-up capital of ₹10 crore, Current Trust Funds under the trusteeship management of NCGTC: 1) Credit Guarantee Fund for Skill Development (CGFSD)

• Guarantees for Skill Development Loans by the member banks of IBA up to ₹ 1.5 lakh extended without collateral or third-party guarantee and the fund has a Target of 10- 20 lakh loans to be guaranteed in a year. 2) Credit Guarantee Fund for Education loans (CGFEL)

• Guarantees for Education Loans by the member banks of IBA up to ₹ 7.5 lakh extended without collateral or third-party guarantee and the fund has a Target of 10 lakh loans to be guaranteed in a year. 3) Credit Guarantee Fund for Factoring (CGFF)

• Guarantees for domestic factored debts of MSMEs.

www.insightsonindia.com 19 Insights IAS INSTA 75 Days REVISION PLAN for Prelims 2020 - InstaTests

4) Credit Guarantee Fund for Micro Units (CGFMU)

• Guarantees for loans up to the specified limit(currently ₹ 10Lakh) sanctioned by Banks / NBFCs / MFIs / other financial intermediaries engaged in providing credit facilities to eligible micro units. Further, Overdraft loan amount of ₹ 5,000/- sanctioned under PMJDY accounts shall also be eligible to be covered under Credit guarantee Fund. 5) Credit Guarantee Fund for Standup India (CGFSI)

• Guarantees for credit facilities of over ₹ 10 lakh & upto ₹ 100 lakh sanctioned by the eligible lending institutions, under the Stand Up India Scheme(SC/ST/Women for setting up Greenfield enterprises).

21. Consider the following statements regarding Koundinya Wildlife Sanctuary 1. It is the only sanctuary in Telangana with a population of Asian elephants. 2. It is covered by southern tropical dry deciduous and thorn forests. Which of the statements given above is/are correct? (a) 1 only (b) 2 only (c) Both 1 and 2 (d) Neither 1 nor 2

Solution: B

Koundinya Wildlife Sanctuary

• Kaundinya Wildlife Sanctuary is a wildlife sanctuary and an elephant reserve situated in Andhra Pradesh, India. It is the only sanctuary in Andhra Pradesh with a population of Asian elephants, which migrated after 200 years from neighbouring regions. • The sanctuary is covered by southern tropical dry deciduous and thorn forests. Some of the important flora consists of Albiziaamara, Acacia, Lagerstroemia, Ficus, bamboo, and a species which is a regeneration of Santalum album. • The sanctuary is situated in region where the Kolar Plateau ends and slopes down into the plains of Tamil Nadu creating many valleys and ghats • The sanctuary is primarily an elephant reserve and is home to about 78 Indian elephants.

www.insightsonindia.com 20 Insights IAS INSTA 75 Days REVISION PLAN for Prelims 2020 - InstaTests

22. Consider the following statements regarding Purchasing Managers’ Index (PMI) 1. It is released by Reserve Bank of India. 2. It is an indicator of business activity, both in the manufacturing and services sectors. 3. A figure above 50 denotes expansion in business activity. Anything below 50 denotes contraction. Which of the statement above is/are correct? (a) 2 only (b) 2 and 3 only (c) 1 and 3 only (d) 1, 2 and 3

Solution: B

Purchasing Managers’ Index (PMI) by Japanese firm Nikkei

• It is an indicator of business activity – both in the manufacturing and services sectors. • The purpose of the PMI is to provide information about current business conditions to company decision makers, analysts and purchasing managers. • It is a survey-based measures that asks the respondents about changes in their perception of some key business variables from the month before. • A figure above 50 denotes expansion in business activity. Anything below 50 denotes contraction.

23. Consider the following statements regarding Interpol’s Red Notices 1. It has been issued by Interpol general secretariat 2. It is an international arrest warrant 3. Interpol can compel any member country to arrest an individual subject to red notice Which of the statements given above is/are correct? (a) 1 and 3 only (b) 2 and 3 only (c) 1 only (d) 1, 2 and 3

www.insightsonindia.com 21 Insights IAS INSTA 75 Days REVISION PLAN for Prelims 2020 - InstaTests

Solution: C

Red Notice:

• A Red Notice is a request to law enforcement worldwide to locate and provisionally arrest a person pending extradition, surrender, or similar legal action. It contains two main types of information:

• Information to identify the wanted person, such as their name, date of birth, nationality, hair and eye colour, photographs and fingerprints if available. • Information related to the crime they are wanted for, which can typically be murder, rape, child abuse or armed robbery. • Red Notices are published by INTERPOL at the request of a member country, and must comply with INTERPOL’s Constitution and Rules. • A Red Notice is not an international arrest warrant • Interpol cannot compel any member country to arrest an individual subject to red notice.

24. Consider the following statements regarding Ways and Means Advances (WMA) 1. They are temporary loan facilities provided by RBI to the government to enable it to meet temporary mismatches between revenue and expenditure. 2. The rate of interest is the same as the repo rate, while the tenure is three months. Which of the statement above is/are correct? (a) 1 only (b) 2 only (c) Both 1 and 2 (d) Neither 1 nor 2

Solution: C

• The Reserve Bank of India (RBI) has announced a 60% increase in the Ways and Means Advances (WMA) limit of state governments over and above the level as on March 31, with a view to enabling them “to undertake COVID-19 containment and mitigation efforts” and “to better plan their market borrowings”.

www.insightsonindia.com 22 Insights IAS INSTA 75 Days REVISION PLAN for Prelims 2020 - InstaTests

Significance of this move:

• The increased limit comes at a time when government expenditure is expected to rise as it battles the fallout of a spreading Coronavirus. The availability of these funds will give government some room to undertake short term expenditure over and above its long term market borrowings. What are Ways and Means Advances?

• They are temporary loan facilities provided by RBI to the government to enable it to meet temporary mismatches between revenue and expenditure. • The government makes an interest payment to the central bank when it borrows money. • The rate of interest is the same as the repo rate, while the tenure is three months. • The limits for WMA are mutually decided by the RBI and the Government of India. • They aren’t a source of finance per se. Section 17(5) of the RBI Act, 1934 authorises the central bank to lend to the Centre and state governments subject to their being repayable “not later than three months from the date of the making of the advance”. Background:

• The WMA scheme for the Central Government was introduced on April 1, 1997, after putting an end to the four-decade old system of ad-hoc (temporary) Treasury Bills to finance the Central Government deficit. What if the government needs extra money for extra time?

• When the WMA limit is crossed the government takes recourse to overdrafts, which are not allowed beyond 10 consecutive working days. • The interest rate on overdrafts would be 2 percent more than the repo rate. Types of WMA: There are two types of Ways and Means Advances — normal and special. 1. Special WMA or Special Drawing Facility is provided against the collateral of the government securities held by the state. After the state has exhausted the limit of SDF, it gets normal WMA. The interest rate for SDF is one percentage point less than the repo rate. 2. The number of loans under normal WMA is based on a three-year average of actual revenue and capital expenditure of the state. What are the existing WMA limits and overdraft conditions?

• For the Centre, the WMA limit during the first half of 2020-21 (April-September) has been fixed at Rs 120,000 crore. This is 60% higher than the Rs 75,000 crore limit for the same period of 2019-20. The limit for the second half of the last fiscal (October- March) was Rs 35,000 crore.

www.insightsonindia.com 23 Insights IAS INSTA 75 Days REVISION PLAN for Prelims 2020 - InstaTests

• For the states, the aggregate WMA limit was Rs 32,225 crore till March 31, 2020. On April 1, the RBI announced a 30% hike in this limit, which has now been enhanced to 60%, taking it to Rs 51,560 crore. The higher limit will be valid till September 30. • The central bank, on April 7, also extended the period for which a state can be in overdraft from 14 to 21 consecutive working days, and from 36 to 50 working days during a quarter.

25. Consider the following statements regarding Institute of Chartered Accountants of India 1. It is a statutory body 2. It functions under the administrative control of the Ministry of Corporate Affairs 3. It is aimed at regulating the profession of Chartered Accountancy in the country. Which of the statements given above is/are correct? (a) 2 and 3 only (b) 1 only (c) 1 and 2 only (d) 1, 2 and 3

Solution: D

Institute of Chartered Accountants of India (ICAI)

• The Institute of Chartered Accountants of India (ICAI) is a statutory body established by an Act of Parliament of India, ‘The Chartered Accountants Act, and 1949′, to regulate the profession of Chartered Accountancy in India. • ICAI is the second largest professional Accounting & Finance body in the world. • ICAI is the only licensing cum regulating body of the financial audit and accountancy profession in India. • It recommends the accounting standards to be followed by companies in India to National Advisory Committee on Accounting Standards (NACAS). • ICAI is solely responsible for setting the Standards on Auditing (SAs) to be followed in the audit of financial statements in India. • ICAI is one of the founder members of the International Federation of Accountants (IFAC), South Asian Federation of Accountants (SAFA), and Confederation of Asian and Pacific Accountants (CAPA). • The Institute functions under the administrative control of the Ministry of Corporate Affairs.

www.insightsonindia.com 24 Insights IAS INSTA 75 Days REVISION PLAN for Prelims 2020 - InstaTests

DAY – 66 (InstaTest-66)

26. Consider the following statements regarding Armed Forces (Special Powers) Act (AFSPA). 1. The Central Government, or the Governor of the State or administrator of the Union Territory can declare the whole or part of the State or Union Territory as a disturbed area. 2. Presently, it is operational in the entire States of Assam, Nagaland and Manipur only 3. Armed can use force or even open fire after giving due warning if they feel a person is in contravention of the law Which of the statements given above is/are correct? (a) 1 and 3 only (b) 2 and 3 only (c) 1 only (d) 1, 2 and 3

Solution: A

Armed Forces (Special Powers) Act (AFSPA) gives armed forces the power to maintain public order in “disturbed areas”. They have the authority to prohibit a gathering of five or more persons in an area, can use force or even open fire after giving due warning if they feel a person is in contravention of the law

• The Central Government, or the Governor of the State or administrator of the Union Territory can declare the whole or part of the State or Union Territory as a disturbed area • Presently, AFSPA, 1958, is operational in the entire States of Assam, Nagaland, Manipur (except Imphal Municipal area), three districts namely Tirap, Changlang and Longding of Arunachal Pradesh and the areas falling within the jurisdiction of the eight police stations in the districts of Arunachal Pradesh, bordering the State of Assam. • The notification declaring Manipur and Assam as “Disturbed Areas’ has been issued by the State governments. For Nagaland, the notification is issued by the MHA. • It is a suo-motto declaration can be made by the Central government, however, it is desirable that the state government should be consulted by the central government before making the declaration.

www.insightsonindia.com 25 Insights IAS INSTA 75 Days REVISION PLAN for Prelims 2020 - InstaTests https://www.thehindu.com/news/national/other-states/afspa-extended-in- nagaland/article30434509.ece

27. Consider the following statements regarding National Sports Development Fund (NSDF) 1. It was established in 1998 under the Charitable Endowments Act 1890. 2. The purpose of creation is to impart momentum and flexibility to assisting the cause of sports 3. The Fund is managed by a Council and Union Minister in charge of Youth Affairs and Sports is the Chairperson of the Council. Which of the statements given above is/are correct? (a) 1 and 2 only (b) 2 and 3 only (c) 1 and 3 only (d) 1, 2 and 3

Solution: D

NATIONAL SPORTS DEVELOPMENT FUND

• National Sports Development Fund (NSDF) was established in 1998 under the Charitable Endowments Act 1890; • It was notified by Government of India in November, 1998; • Purpose of creation is to impart momentum and flexibility to assisting the cause of sports; • The Fund helps sportspersons excel by providing them opportunities to train under coaches of international repute with technical, scientific and psychological support and giving them exposure to international competitions; • The Fund also provides financial assistance for development of infrastructure and other activities for promotion of sports; • Role of the Fund is supplementary to the overall policy and activities of the Department of Sports in achieving excellence in sports. OBJECTIVES OF NSDF

• To administer and apply the moneys of the Fund for promotion of sports in general and specific sports disciplines and individual sports persons in particular for achieving excellence at the National and of International level; • To impart special training and coaching in relevant sports disciplines to sports persons, coaches and sports specialists;

www.insightsonindia.com 26 Insights IAS INSTA 75 Days REVISION PLAN for Prelims 2020 - InstaTests

• To construct and maintain infrastructure for promotion of sports and games; • To supply sports equipments to organizations and individuals for promotion of sports and games; • To identify problems and take up research and development studies for providing support to excellence in sports; • To promote international cooperation, in particular, exchanges which may promote the development of sports; • To provide low-interest or interest-free loans for projects and activities related to any of the aforesaid objects. MANAGEMENT AND ADMINISTRATION OF NSDF Council of NSDF

• The Fund is managed by a Council constituted by the Central Government. Union Minister in charge of Youth Affairs and Sports is the Chairperson of the Council. Members of the Council include senior Officers of the Department of Sports, Chairman & Managing Directors of Private and Public Sector Companies / Corporations, representatives of Sports Promotion Boards, etc. • The Council decides all policy matters relating to the Fund.

28. Consider the following statements regarding exchange rate 1. The Real Effective Exchange Rate (REER) of the rupee is a weighted average of exchange rates before the currencies of India’s major trading partners. 2. When the weight of inflation is adjusted with the REER, we get the Nominal Effective Exchange Rate (REER) of the rupee. Which of the statements given above is/are correct? (a) 1 only (b) 2 only (c) Both 1 and 2 (d) Neither 1 nor 2

Solution: D

NEER

• The Nominal Effective Exchange Rate (NEER) of the rupee is a weighted average of exchange rates before the currencies of India’s major trading partners.

www.insightsonindia.com 27 Insights IAS INSTA 75 Days REVISION PLAN for Prelims 2020 - InstaTests

REER

• When the weight of inflation is adjusted with the NEER, we get the Real Effective Exchange Rate (REER) of the rupee. Since inflation has been on the higher side in recent months, the REER of the rupee has been more against it than the NEER.

29. Consider the following statements regarding Skills Build Platform 1. It is skill building programme for migrant workers. 2. It is launched by Ministry of Labour with active co-operation by skill ministry Which of the statements given above is/are correct? (a) 1 only (b) 2 only (c) Both 1 and 2 (d) Neither 1 nor 2

Solution: D

• Ministry of Skill Development & Entrepreneurship launches Skills Build platform in Collaboration with IBM. Key facts:

• Launched by Directorate General of Training (DGT), under the aegis of Ministry of Skill Development & Entrepreneurship (MSDE). • As part of the programme, a two-year advanced diploma in IT, networking and cloud computing, co-created and designed by IBM, will be offered at the Industrial Training Institutes (ITIs) & National Skill Training Institutes (NSTIs). • The platform will be extended to train ITI & NSTI faculty on building skills in Artificial Intelligence (AI). Significance of the programme:

• The digital platform will provide a personal assessment of the cognitive capabilities and personality via My Inner Genius to the students. • They will then learn foundational knowledge about digital technologies, as well as professional skills such as resume-writing, problem solving and communication. • Students will also receive recommendations on role-based education for specific jobs that include technical and professional learning. • This initiative is part of IBM’s global commitment to create a job-ready workforce and to build the next generation of skills needed for new collar careers.

www.insightsonindia.com 28 Insights IAS INSTA 75 Days REVISION PLAN for Prelims 2020 - InstaTests https://www.insightsonindia.com/2019/11/05/skills-build-platform/

30. Consider the following statements regarding Hybrid Annuity Model (HAM) 1. The HAM is a mix between the existing two models – BOT Annuity and EPC. 2. As per the design, the government will contribute to 60% of the project cost in the first five years through annual payments (annuity). 3. Revenue collection would be the responsibility of the Government. Which of the statements given above is/are correct? (a) 1 and 2 only (b) 3 only (c) 1 and 3 only (d) 1, 2 and 3

Solution: C

Hybrid Annuity Model (HAM) has been introduced by the Government to revive PPP (Public Private Partnership) in highway construction in India.

• At present, three different models –PPP Annuity, PPP Toll and EPC (Engineering, Procurement and Construction) were followed by the government while adopting private sector participation. Features of HAM

• By features the HAM is a mix between the existing two models – BOT Annuity and EPC. Hence to understand the HAM, the basic features of the existing PPP models are elucidated first. a) The Build Operate and Transfer (BOT) Annuity Model

• Under BOT annuity, a developer builds a highway, operates it for a specified duration and transfers it back to the government. The government starts payment to the developer after the launch of commercial operation of the project. Payment will be made on a six month basis. b) BOT Toll Model

• In this toll based BOT model, a road developer constructs the road and he is allowed to recover his investment through toll collection. This toll collection will be over a long period which is nearly 30 years in most cases. There is no government payment to the developer as he earns his money invested from tolls.

www.insightsonindia.com 29 Insights IAS INSTA 75 Days REVISION PLAN for Prelims 2020 - InstaTests c) Engineering, Procurement and Construction (EPC) Model

• Under this model, the cost is completely borne by the government. Government invites bids for engineering knowledge from the private players. Procurement of raw material and construction costs are met by the government. The private sector’s participation is minimum and is limited to the provision of engineering expertise. A difficulty of the model is the high financial burden for the government. The Hybrid Annuity Model (HAM):

• In India, the new HAM is a mix of BOT Annuity and EPC models. As per the design, the government will contribute to 40% of the project cost in the first five years through annual payments (annuity). The remaining payment will be made on the basis of the assets created and the performance of the developer. • Here, hybrid annuity means the first 40% payment is made as fixed amount in five equal installments whereas the remaining 60% is paid as variable annuity amount after the completion of the project depending upon the value of assets created. • As the government pays only 40%, during the construction stage, the developer should find money for the remaining amount. • Here, he has to raise the remaining 60% in the form of equity or loans. • There is no toll right for the developer. Under HAM, Revenue collection would be the responsibility of the National Highways Authority of India (NHAI).

31. Consider the following statements regarding Invest India 1. It is a non-profit venture under the Department for Promotion of Industry and Internal Trade, Ministry of Commerce and Industry. 2. It is the National Investment Promotion and Facilitation Agency of India and act as the first point of reference for investors in India. 3. The current shareholding pattern is 49 % of Industry Associations and the remaining 51% of Central and State Governments. Which of the statements given above is/are correct? (a) 1 and 2 only (b) 2 only (c) 1 and 3 only (d) 1, 2 and 3

Solution: A

www.insightsonindia.com 30 Insights IAS INSTA 75 Days REVISION PLAN for Prelims 2020 - InstaTests

• Invest India, set up in 2009, is a non-profit venture under the Department for Promotion of Industry and Internal Trade, Ministry of Commerce and Industry, Government of India. • We are the National Investment Promotion and Facilitation Agency of India and act as the first point of reference for investors in India. • As the national investment promotion and facilitation agency, Invest India focuses on sector-specific investor targeting and development of new partnerships to enable sustainable investments in India. In addition to a core team that focuses on sustainable investments, Invest India also partners with substantial investment promotion agencies and multilateral organizations. Invest India also actively works with several Indian states to build capacity as well as bring in global best practices in investment targeting, promotion and facilitation areas. • Invest India was formed in 2009 under Section 25 of the Companies Act 1956 for promotion of foreign investment with 49% equity of the then Department of Industrial Policy and Promotion, Ministry of Commerce and Industry and 51% shareholding by FICCI. The current shareholding pattern of Invest India is 51 % of Industry Associations (i.e. 17% each of FICCI, CII & NASSCOM) and the remaining 49% of Central and 19 State Governments. • Department for Promotion of Industry and Internal Trade conducts regular review of the performance of Invest India. Further, the Board of Directors, under chairmanship of Secretary, Department for Promotion of Industry and Internal Trade, including nominees from Government of India, FICCI, CII and NASSCOM manages and oversee the overall operations, direction and strategy of the company. Regular meetings of the Board of Directors are held to monitor the operational and overall performance of Invest India.

32. Agreement on Reciprocal Logistics Support (ARLS), sometime seen in the news, is a defence agreement between which of the following countries? (a) India and United States (b) India and Japan (c) India and Russia (d) India and France

Solution: C

www.insightsonindia.com 31 Insights IAS INSTA 75 Days REVISION PLAN for Prelims 2020 - InstaTests

Agreement on Reciprocal Logistics Support (ARLS)

• India and Russia are finalizing a defence agreement that will simplify interoperability and enable military platforms to receive support and supplies across bases in both nations. • This will be beneficial for the Indian Navy, which has a large number of Russian origin ships that will get access to Russian ports for supplies and refueling. It would be crucial for joint exercises. • It is an arrangement that will allow access to India and Russia, to each other’s military facilities for supplies and fuel. https://www.thehindu.com/news/national/india-russia-to-conclude-mutual-logistics- agreement/article29881623.ece

33. Consider the following statements regarding Indian Ports 1. Major Ports are under the Union List while the Non-Major Ports are under the State List of the Constitution of India. 2. Major Ports are under the administrative control of Government of India while the Non-major ports are governed by the respective Maritime State Governments. 3. Major ports handle more than 50% of sea-borne traffic. Which of the statements given above is/are correct? (a) 2 only (b) 2 and 3 only (c) 1 and 3 only (d) 1, 2 and 3

Solution: B

• Indian Ports are broadly classified as Major Ports and Non-Major Ports. The Major Ports are under the Union List while the Non-Major Ports are under the Concurrent List of the Constitution of India. Hence, Major Ports are under the administrative control of Government of India while the Non-major ports are governed by the respective Maritime State Governments. • Major Ports are defined in Section 3(8) of the Indian Ports Act 1908 to mean any port which the Central Government may by notification in the Official Gazette declare, or may under any law for the time being in force have declared, to be a major port. • India has 12 major ports, which handle about 58% of sea-borne traffic. These are Kolkata (including Dock Complex at Haldia), , Visakhapatnam, Chennai, V.O. www.insightsonindia.com 32 Insights IAS INSTA 75 Days REVISION PLAN for Prelims 2020 - InstaTests

Chidambaranar (Tuticorin), Cochin, New Mangalore, Mormugao, Jawaharlal Nehru Port Trust (JNPT), Mumbai, Kandla and Ennore. Of these, Ennore Port Ltd is a company and the remaining 11 are Port Trusts, governed by the provisions of the Major Port trusts Act, 1963. • Under the Indian Ports Act, 1908, the Government has declared the Port Blair Port with its territorial jurisdiction over all ports of Andaman & Nicobar Islands, – as a major port w.e.f. 1 June, 2010. All major provisions of the Major Port Trusts Act, 1963 has become applicable to the major port of Port Blair from 1 June, 2010. Though Government has intended it to be the next major port, no further action has been taken on the same. • India has around 200 Non-Major ports, which handle about 42% of sea-borne traffic. • To allow the competitive market forces to play a greater role in determination of tariff at Major Port Trusts, the Government has issued two policy guidelines viz. Guidelines for Determination of Tariff for Projects at Major Ports, 2013 and Guidelines for Determination of Tariff for Major Port Trusts, 2015. These guidelines impart flexibility to the PPP operators in the Major Ports and Major Ports owned terminals in determining their tariff, subject to a ceiling rate.

34. Consider the following statements regarding Local Area Banks (LABs) 1. They provide services in a limited area of operation, i.e., primarily in rural and semi-urban areas, comprising three contiguous districts. 2. LABs were required to have a minimum capital of Rs. 50 crores. Which of the statements given above is/are correct? (a) 1 only (b) 2 only (c) Both 1 and 2 (d) Neither 1 nor 2

Solution: A

• The Local Area Banks (LABs) are small private banks, conceived as low-cost structures which would provide efficient and competitive financial intermediation services in a limited area of operation, i.e., primarily in rural and semi-urban areas, comprising three contiguous districts. • LABs were set up to enable the mobilization of rural savings by local institutions and, at the same time, to make them available for investments in the local areas.

www.insightsonindia.com 33 Insights IAS INSTA 75 Days REVISION PLAN for Prelims 2020 - InstaTests

• LABs were created following an announcement made by the then Finance Minister in the Union Budget in August 1996. RBI issued guidelines for setting up of Local Area Banks (LABs) vide its Press Release dated August 24, 1996. • LABs were required to have a minimum capital of Rs. 5 crores. • The promoters of the bank may comprise of private individuals, corporate entities, trusts and societies with a minimum capital contribution of Rs. 2 crores. • The area of operation of LAB is limited to a maximum of three geographically contiguous districts and are allowed to open branches only in its area of operation. • Since LABs are being set up in district towns, their activities are focused on the local customers with lending primarily to agriculture and allied activities, small scale industries, agro-industrial activities, trading activities and the non-farm sector. LABs are also required to observe the priority sector lending targets at 40% of net bank credit (NBC) as applicable to other domestic banks. Within the above target, these banks will adhere to the requirement of lending at least 25% of their priority sector deployments (10% of NBC) to the weaker sections. • Around 5 LABs were licensed by 2002 under Section 22 of the Banking Regulation Act 1949 from amongst hundreds of applicants. Presently, four LABs are functioning satisfactorily. • In 2014, RBI has permitted LABs to be converted into small finance banks subject to them meeting the prescribed eligibility criteria.

35. Consider the following statements regarding National Nutrition Survey 1. The survey for the first time proved the coexistence of obesity and under nutrition. 2. Around 10% of children in the age group of 5 to 9 years and adolescents in the age group 10 to 19 years are pre – diabetic Which of the statements given above is/are correct? (a) 1 only (b) 2 only (c) Both 1 and 2 (d) Neither 1 nor 2

Solution: C

National Nutrition Survey

• With the help of UNICEF, the Ministry of Health and Family Welfare recently conducted the first-ever comprehensive National Nutrition Survey.

www.insightsonindia.com 34 Insights IAS INSTA 75 Days REVISION PLAN for Prelims 2020 - InstaTests

• The survey recorded malnutrition that included micronutrient deficiencies and details of non-communicable diseases such as diabetes, hypertension, cholesterol and kidney function in children and adolescents • The survey for the first time proved the coexistence of obesity and under nutrition. • Around 10% of children in the age group of 5 to 9 years and adolescents in the age group 10 to 19 years are pre – diabetic. The survey for the first time proved the coexistence of obesity and under nutrition. • One in five children in the age group 5 to 9 years were stunted https://www.insightsonindia.com/2019/10/01/national-nutrition-survey/

36. Which of the following is/are considered as minor minerals in India? 1. Stones used for making household utensils 2. Bentonite 3. Shale when used for building material 4. Mica Select the correct answer using the code given below: (a) 1, 2 and 3 only (b) 2, 3 and 4 only (c) 1, 2 and 4 only (d) 1, 2, 3 and 4

Solution: D

• In India, the minerals are classified as minor minerals and major minerals. • According to section 3(e) of the Mines and Minerals (Development and Regulation) Act, 1957 “Minor Minerals” means building stones, gravel, ordinary clay, ordinary sand other than sand used for prescribed purposes, and any other mineral which the Central Government may, by notification in the Official Gazette, declare to be a minor mineral. (For the purposes of this Act, the word “minerals” includes all minerals except mineral oils- natural gas and petroleum) • Major minerals are those specified in the first schedule appended in the Mines and Minerals (Development and Regulation) Act, 1957 (MMDR Act 1957) and the common major minerals are Lignite, Coal, Uranium, iron ore, gold etc. It may be noted that there is no official definition for “major minerals” in the MMDR Act. Hence, whatever is not declared as a “minor mineral” may be treated as the major mineral. • The major-minor classification has nothing to do with the quantum /availability of these minerals, though it is correlated with the relative value of these minerals.

www.insightsonindia.com 35 Insights IAS INSTA 75 Days REVISION PLAN for Prelims 2020 - InstaTests

Further, this classification is based more on their end use, rather than level of production, level of mechanization, export and import etc. (eg. Sand can be a major mineral or a minor mineral depending on where it is used; same is the case for limestone.) • India produces as many as 88 minerals which include 4 fuel minerals, 3 atomic minerals, 26 metallic & non-metallic minerals and 55 minor minerals (including building and other materials and the recently notified 31 additional minerals). • The central government has the power to notify “minor minerals” under section 3 (e) of the MMDR Act, 1957. On the other hand, as per Section 15 of the MMDR Act, 1957 State Governments have complete powers for making Rules for grant of concessions in respect of extraction of minor minerals and levy and collection of royalty on minor minerals. In addition to the minor minerals specified in Section 3(e) of the MMDR Act, the Central Government has declared the following minerals as minor minerals:

• boulder, • shingle, • chalcedony pebbles used for ball mill purposes only, • lime shell, kankar and limestone used in kilns for manufacture of lime used as building material, • murrum, • brick-earth, • fuller’s earth, • bentonite, • road metal, • reh-matti, • slate and shale when used for building material, • marble, • stone used for making household utensils, • quartzite and sandstone when used for purposes of building or for making road metal and household utensils, • saltpeter and • ordinary earth (used or filling or leveling purposes in construction or embankments, roads, railways, building). Mica; Ochre; Pyrophyllite; Quartz; Quartzite; Sand (Others); Shale;

www.insightsonindia.com 36 Insights IAS INSTA 75 Days REVISION PLAN for Prelims 2020 - InstaTests

37. Consider the following statements regarding Mumbai Inter-Bank Offer Rate (MIBOR) and Mumbai Inter-Bank Bid Rate (MIBID) 1. They are the benchmark rates at which Indian banks lend and borrow money to each other. 2. MIBID is the rate at which banks would like to borrow from other banks and MIBOR is the rate at which banks are willing to lend to other banks. Which of the statements given above is/are correct? (a) 1 only (b) 2 only (c) Both 1 and 2 (d) Neither 1 nor 2

Solution: C

• Mumbai Inter-Bank Offer Rate (MIBOR) and Mumbai Inter-Bank Bid Rate (MIBID) are the benchmark rates at which Indian banks lend and borrow money to each other. The bid is the price at which the market would buy and the offer (or ask) is the price at which the market would sell. These rates reflect the short-term funding costs of major banks. In other words, MIBOR reflects the price at which short term funds are made available to participating banks. • MIBID is the rate at which banks would like to borrow from other banks and MIBOR is the rate at which banks are willing to lend to other banks. Contrary to general perception, MIBID is not the rate at which banks attract deposits from other banks. • MIBOR is the Indian version of London Interbank Offer Rate (LIBOR). MIBOR is fixed for overnight to 3 month long funds and these rates are published every day at a designated time. Of the above tenors, the overnight MIBOR is the most widely used one which is used for pricing and settlement of Overnight Index Swaps (OIS). Corporates use the OIS for hedging their interest rate risks. The MIBID/MIBOR rate is also used as a bench mark rate for majority of deals struck for Interest Rate Swaps (IRS), Forward Rate Agreements (FRA), Floating Rate Debentures and Term Deposits. The aggregate amount of outstanding interbank/Primary Dealers (PD) notional principal referenced to MIBOR remained at INR 16,847.6 billion as on October 31, 2013 Financial Benchmarks

• MIBOR, MIBID etc. are all financial benchmarks.

www.insightsonindia.com 37 Insights IAS INSTA 75 Days REVISION PLAN for Prelims 2020 - InstaTests

38. Consider the following statements regarding School Education Quality Index (SEQI) 1. Kerala has emerged on top among 20 large states in terms of quality of school education. 2. It aims to bring an ‘outcomes’ focus to education policy by providing States and UTs with a platform to identify their strengths and weaknesses 3. All seven union territories have shown significant decline in their overall performance scores Which of the statements given above is/are correct? (a) 1 only (b) 1 and 3 only (c) 1 and 2 only (d) 2 and 3 only

Solution: C

School Education Quality Index (SEQI): By NITI Aayog

• The School Education Quality Index (SEQI) was developed to evaluate the performance of States and Union Territories (UTs) in the school education sector. The index aims to bring an outcomes focus to education policy by providing States and UTs with a platform to identify their strengths and weaknesses and undertake requisite course corrections or policy interventions. In line with NITI Aayog’s mandate to foster the spirit of competitive and cooperative federalism, the index strives to facilitate the sharing of knowledge and best practices across States and UTs. • It aims to bring an ‘outcomes’ focus to education policy by providing States and UTs with a platform to identify their strengths and weaknesses and undertake requisite course corrections or policy interventions. • The index is developed through a collaborative process, including key stakeholders such as Ministry of Human Resource and Development (MHRD), the World Bank and sector experts. • It consists of 30 critical indicators that assess the delivery of quality education. • Kerala has emerged on top among 20 large states in terms of quality of school education, followed by Rajasthan and Karnataka, [while the most-populous Uttar Pradesh was ranked at the bottom position during 2016-17] • The overall performance has declined in Karnataka and Uttarakhand. • All seven union territories have shown an improvement in their overall performance scores. https://niti.gov.in/content/school-education-quality-index

www.insightsonindia.com 38 Insights IAS INSTA 75 Days REVISION PLAN for Prelims 2020 - InstaTests

39. Consider the following statements regarding Commodities transaction tax (CTT) 1. Commodities transaction tax (CTT) is a tax similar to Securities Transaction Tax (STT), levied in India, on transactions done on the domestic commodity derivatives exchanges. 2. The concept of CTT was first introduced in the Union Budget 2018-19. 3. All agricultural commodities are exempted from CTT. Which of the statements given above is/are correct? (a) 1 only (b) 2 and 3 only (c) 1 and 3 only (d) 1, 2 and 3

Solution: A

• Commodities transaction tax (CTT) is a tax similar to Securities Transaction Tax (STT), levied in India, on transactions done on the domestic commodity derivatives exchanges. In, Finance Act, 2016 it was stipulated that transactions carried out in a recognized commodity exchange located in an International Financial Center, where the payments are carried out in terms of foreign currency, would be exempt from the payment of CTT. • Globally, commodity derivatives are also considered as financial contracts. Hence CTT can also be considered as a type of financial transaction tax. • The concept of CTT was first introduced in the Union Budget 2008-09 (para 179 of the Budget Speech). The Government had then proposed to impose a commodities transaction tax (CTT) of 0.017% (equivalent to the rate of equity futures at that point of time). However, it was withdrawn subsequently as the market was nascent then and any imposition of transaction tax might have adversely affected the growth of organized commodities derivatives markets in India. This has helped Indian commodity exchanges to grow to global standards (MCX is the world’s number 3 commodity exchange; Globally, MCX is No. 1 in Gold and Silver, No. 2 in Natural gas and No. 3 in Crude Oil) • In the Union Budget 2013-14 (para 149 of the Budget Speech) CTT has been re- introduced, however, only for non-agricultural commodity futures at the rate of 0.01% (which is equivalent to the rate of equity futures). Along with this, transactions in commodity derivatives have been declared to be made non-speculative; and hence for traders in the commodity derivative segment, any losses arising from such transactions can be set off against income from any other source (similar provisions are applicable for the securities market transactions). • A separate provision for CTT has been made in the Finance Act, 2013 (Chapter VII).

www.insightsonindia.com 39 Insights IAS INSTA 75 Days REVISION PLAN for Prelims 2020 - InstaTests

• The CTT rules were notified by Department of Revenue, Ministry of Finance on 19 June 2013 (Notification No. 46 of 2013; S.O. 1769 (E)), with effect from 1 July 2013. As per the notification, only 23 agricultural commodities were exempted from CTT. However, in February 2015 a revised list of 61 commodities was notified including certain commodities where trading is currently not taking place. Like STT, the commodity exchanges have been entrusted to collect CTT on behalf of Government of India. • Like all financial transaction taxes, CTT aims at discouraging excessive speculation, which is detrimental to the market and to bring parity between securities market and commodities market such that there is no tax / regulatory arbitrage.

40. Consider the following statements regarding Gini index or Gini coefficient 1. It is often used as a gauge of economic inequality, measuring income distribution or, less commonly, wealth distribution among a population. 2. The coefficient ranges from 0 (or 0%) to 1 (or 100%), with 0 representing perfect equality and 1 representing perfect inequality. 3. The Gini index is often represented graphically through the Lorenz curve. Which of the statements given above is/are correct? (a) 1 and 2 only (b) 2 and 3 only (c) 1 and 3 only (d) 1, 2 and 3

Solution: D

• The Gini index or Gini coefficient is a statistical measure of distribution developed by the Italian statistician Corrado Gini in 1912. It is often used as a gauge of economic inequality, measuring income distribution or, less commonly, wealth distribution among a population. The coefficient ranges from 0 (or 0%) to 1 (or 100%), with 0 representing perfect equality and 1 representing perfect inequality. Values over 1 are theoretically possible due to negative income or wealth. • The Gini index is a simple measure of the distribution of income across income percentiles in a population. • A higher Gini index indicates greater inequality, with high income individuals receiving much larger percentages of the total income of the population. • Global inequality as measured by the Gini index increased over the 19th and 20th centuries, but has declined in more recent years.

www.insightsonindia.com 40 Insights IAS INSTA 75 Days REVISION PLAN for Prelims 2020 - InstaTests

• Because of data and other limitations, the Gini index may overstate income inequality and can obscure important information about income distribution. • The Gini index is often represented graphically through the Lorenz curve, which shows income (or wealth) distribution by plotting the population percentile by income on the horizontal axis and cumulative income on the vertical axis. The Gini coefficient is equal to the area below the line of perfect equality (0.5 by definition) minus the area below the Lorenz curve, divided by the area below the line of perfect equality. In other words, it is double the area between the Lorenz curve and the line of perfect equality.

41. Consider the following statements regarding Rashtriya Aavishkar Abhiyan (RAA) 1. It has been launched under Atal Innovation Mission 2. It will target students in the age group of 6 – 18 years. 3. It aims to encourage children towards learning Science and Mathematics. Which of the statements given above is/are correct? (a) 1 only (b) 2 and 3 only (c) 1 and 3 only (d) 1, 2 and 3

Solution: B

Rashtriya Aavishkar Abhiyan (RAA):

• It was launched in 2015. • Ministry of Human Resource Development has launched the Rashtriya Avishkar Abhiyan (RAA). • It is a convergent framework across School Education and Higher Education to motivate children of the age group from 6-18 years in learning Science, Mathematics and Technology through observation, experimentation, inference drawing, model building, etc. both through inside and outside classroom activities and processes. It seeks to create curiosity, excitement and spirit of innovation and exploration amongst school children by encouraging higher education institutions to become Mentoring Institutions and assist secondary and elementary schools in the study of Science and Mathematics. • Major interventions under RAA provided under Integrated scheme for School Education – Samagra Shiksha, include conduct of Science Exhibition, Book Fair, Quiz Competition, exposure/study visits for students to Higher Education institutions, participation of students in Inter-school/State/National level Science & Maths

www.insightsonindia.com 41 Insights IAS INSTA 75 Days REVISION PLAN for Prelims 2020 - InstaTests

Competitions/Olympiads, strengthening of School Science and Mathematics laboratories, use of teaching-learning equipment and material including Digital models and use of technology in Science and Mathematics teaching.

42. Consider the following statements regarding National Bank for Agriculture and Rural Development (NABARD) 1. NABARD today is fully owned by RBI. 2. It was setup based on the recommendation of B. Sivaraman Committee. Which of the statements given above is/are correct? (a) 1 only (b) 2 only (c) Both 1 and 2 (d) Neither 1 nor 2

Solution: B

National Bank for Agriculture and Rural Development (NABARD)

• The importance of institutional credit in boosting rural economy has been clear to the Government of India right from its early stages of planning. Therefore, the Reserve Bank of India (RBI) at the insistence of the Government of India, constituted a Committee to Review the Arrangements For Institutional Credit for Agriculture and Rural Development (CRAFICARD) to look into these very critical aspects. The Committee was formed on 30 March 1979, under the Chairmanship of Shri B. Sivaraman, former member of Planning Commission, Government of India. • The Committee’s interim report, submitted on 28 November 1979, outlined the need for a new organizational device for providing undivided attention, forceful direction and pointed focus to credit related issues linked with rural development. Its recommendation was formation of a unique development financial institution which would address these aspirations and formation of National Bank for Agriculture and Rural Development (NABARD) was approved by the Parliament through Act 61 of 1981. • NABARD came into existence on 12 July 1982 by transferring the agricultural credit functions of RBI and refinance functions of the then Agricultural Refinance and Development Corporation (ARDC). It was dedicated to the service of the nation by the late Prime Minister Smt. Indira Gandhi on 05 November 1982. Set up with an initial capital of Rs.100 crore, its’ paid up capital stood at Rs.12,580 crore as on 31 March 2019. Consequent to the revision in the composition of share capital between Government of India and RBI, NABARD today is fully owned by Government of India. www.insightsonindia.com 42 Insights IAS INSTA 75 Days REVISION PLAN for Prelims 2020 - InstaTests

43. Consider the following statements regarding CriSidEx 1. It was launched by CRISIL and SIDBI. 2. It is a composite index which measures business sentiment of Micro and Small Enterprises. 3. A scale of 0 (extremely negative) to 100 (extremely positive). Which of the statements given above is/are correct? (a) 1 and 2 only (b) 2 and 3 only (c) 1 and 3 only (d) 1, 2 and 3

Solution: A

About CriSidEx: CRISIL-SIDBI MSE Sentiment Index, or CriSidEx

• It was launched by CRISIL and SIDBI. • CriSidEx is a composite index based on a diffusion index of 8 parameters and measures MSE business sentiment on a scale of 0 (extremely negative) to 200 (extremely positive). CriSidEx will have 2 indices, one for the ‘survey quarter’ and another for the ‘next quarter’ once a trend emerges after few rounds of the survey, providing independent time series data. • Benefits: The crucial benefit of CriSidEx is that its readings will flag potential headwinds and changes in production cycles and thus help improve market efficiencies. And by capturing the sentiment of exporters and importers, it will also offer actionable indicators on foreign trade. Significance of MSME:

• MSME sector is backbone of the economy. It is one of the largest employer in the country and with the vast population where employment either in government or in the large industry itself has limited potential. This is one sector where people not only exhibit their entrepreneurial skills, become part of large value chains but also become job creators in the process. And that is the reason why a bulk of the jobs in manufacturing, trading have been created in this particular sector.

www.insightsonindia.com 43 Insights IAS INSTA 75 Days REVISION PLAN for Prelims 2020 - InstaTests

44. Consider the following statements regarding Leader of Opposition 1. It is a constitutional post 2. He is leader of the largest party that has not less than one-tenth of the total strength of the house 3. He plays a crucial role in appointments of CVC, CBI, CIC, Lokpal. Which of the statements given above is/are correct? (a) 2 and 3 only (b) 3 only (c) 1 and 2 only (d) 1, 2 and 3

Solution: A

Leader of Opposition:

• The LOP is leader of the largest party that has not less than one-tenth of the total strength of the house. • It is a statutory post defined in the Salaries and Allowances of Leaders of Opposition in Parliament Act, 1977. Significance of the office:

• LoP is referred to as the ‘shadow Prime Minister’. • She/he is expected to be ready to take over if the government falls. • The LoP also plays an important role in bringing cohesiveness and effectiveness to the opposition’s functioning in policy and legislative work. • LoP plays a crucial role in bringing bipartisanship and neutrality to the appointments in institutions of accountability and transparency – CVC, CBI, CIC, Lokpal etc.

45. Consider the following statements regarding Business Correspondent 1. Business correspondents are bank representatives and help villagers to open bank accounts. 2. Only retired bank employees can become Business correspondents. 3. BC’s must be a permanent resident of the area in which they propose to operate. Which of the statements given above is/are correct? (a) 1 and 2 only (b) 2 and 3 only

www.insightsonindia.com 44 Insights IAS INSTA 75 Days REVISION PLAN for Prelims 2020 - InstaTests

(c) 1 and 3 only (d) 1, 2 and 3

Solution: C

Business Correspondent (BC)

• Business correspondents are bank representatives. They help villagers to open bank accounts. Business Correspondents get commission from bank for every new account opening, every transaction made via them, every loan-application processed etc. The Business Correspondent carries a mobile device and helps villagers in banking transactions. (Deposit money, take money out of savings account, loans etc.). The villager gives his thumb impression or electronic signature, and gets the money. Eligibility to become a Business Correspondent As per the RBI guidelines, the following entities are eligible for appointment of Business Correspondents (BCS) for banks:

• NGOs/MFls set up under Societies/Trust Acts • Societies registered under Mutually Aided Cooperative Societies Acts or the Cooperative Societies Acts of States • Section 25 companies that are stand alone entities or in which NBFCs, banks, telecom companies and other corporate entities or their holding companies did not have equity holdings in excess of 10 per cent • Post offices • Retired bank employees, • Ex-servicemen • Retired government employees. • Individual kirana / medical / fair price shop owners Individual Public Call Office (PCO) operators • Agents of Small Savings Schemes of Government of India/Insurance Companies Individuals who own petrol pumps • Retired teachers • Authorized functionaries of well run Self Help Groups (SHGs) linked to banks Non deposit taking NBFCs (non-banking finance companies) in the nature of loan companies whose micro finance portfolio is not less than 80 per cent of their loan outstanding in the financially excluded districts as identified by the Committee on Financial Inclusion. RBI has now permitted banks to engage any individual, including those operating Common Service Centres (CSCs) as BC, subject to banks’ comfort level and their carrying out suitable

www.insightsonindia.com 45 Insights IAS INSTA 75 Days REVISION PLAN for Prelims 2020 - InstaTests due diligence as also instituting additional safeguards as may be considered appropriate to minimise the agency risks. Appointment of BCs

• Must be a permanent resident of the area in which they propose to operate. They should be well established, enjoy good reputation and have the confidence of the local people. • The ability of BCs to invest in POS machines and other equipment. In case of individuals Selected as BCS, the criterion is as under: A minimum education qualification of Xth pass. Field Investigation /RCU for verification of residence and dealings, etc. to be conducted. • Credibility check – A/c with any other bank. Should open account with Bank (base branch) • Suitable amount of Security deposit /Bank guarantee based on business Volumes.

46. Consider the following statements regarding International Court of Justice 1. All members of the UN are parties to the statute of the ICJ. 2. Any individuals, non-governmental organizations, corporations or any other private entities can appear before the ICJ. 3. ICJ only has jurisdiction over any dispute, if the both concerned parties submit to the jurisdiction of ICJ. Which of the statements given above is/are correct? (a) 1 and 2 only (b) 2 and 3 only (c) 1 and 3 only (d) 1, 2 and 3

Solution: C

International Court of Justice:

• The International Court of Justice (ICJ) is the principal judicial body of the UN. Established in 1946 to replace the Permanent Court of International Justice, the ICJ mainly operates under the statute of its predecessor, which is included in the UN Charter. • It has two primary functions: to settle legal disputes submitted by States in accordance with established international laws, and to act as an advisory board on issues submitted to it by authorized international organizations.

www.insightsonindia.com 46 Insights IAS INSTA 75 Days REVISION PLAN for Prelims 2020 - InstaTests

• Only countries are eligible to appear before the ICJ, not individuals, non-governmental organisations, corporations or any other private entities. • ICJ only has jurisdiction over any dispute, if the both concerned parties (states) submit to the jurisdiction of ICJ Members of the Court:

• The International Court of Justice is composed of 15 judges elected to nine-year terms of office by the United Nations General Assembly and the Security Council. These organs vote simultaneously but separately. In order to be elected, a candidate must receive an absolute majority of the votes in both bodies. In order to ensure a measure of continuity, one third of the Court is elected every three years. Judges are eligible for re-election. • All members of the UN are parties to the statute of the ICJ, and non-members may also become parties. Who nominates the candidates?

• Every state government, party to the Charter, designates a group who propose candidates for the office of ICJ judges. This group includes four members/jurists of the Permanent Court of Arbitration (machinery which enables arbitral tribunals to be set up as desired and facilitates their work) also picked by the State. Countries not part of the statute follow the same procedure where a group nominates the candidates. • Each group is limited to nominate four candidates, two of whom could be of their nationality. Within a fixed duration set by the Secretary-General, the names of the candidates have to be sent to him/her. What are the qualifications of ICJ judges?

• A judge should have a high moral character. • A judge should fit to the qualifications of appointment of highest judicial officers as prescribed by their respective states or. • A judge should be a juri-consult of recognized competence in international law. The 15 judges of the Court are distributed as per the regions:

• Three from Africa. • Two from Latin America and Caribbean. • Three from Asia. • Five from Western Europe and other states. • Two from Eastern Europe. Independence of the Judges:

• Once elected, a Member of the Court is a delegate neither of the government of his own country nor of that of any other State. Unlike most other organs of international organizations, the Court is not composed of representatives of governments. Members of the Court are independent judges whose first task, before taking up their

www.insightsonindia.com 47 Insights IAS INSTA 75 Days REVISION PLAN for Prelims 2020 - InstaTests

duties, is to make a solemn declaration in open court that they will exercise their powers impartially and conscientiously. • In order to guarantee his or her independence, no Member of the Court can be dismissed unless, in the unanimous opinion of the other Members, he/she no longer fulfils the required conditions. This has in fact never happened.

47. Which of following are major economy traits of Recovery cycle in the business cycle? 1. An upturn in aggregate (total) demand which has to be accompanied by increase in the level of production. 2. As demand goes upward, inflation also moves upward making borrowing cheaper for investors. 3. Production process expands and new investments become attractive. Select the correct answer using the code given below: (a) 1 and 2 only (b) 2 and 3 only (c) 1 and 3 only (d) 1, 2 and 3

Solution: D

Recovery An economy tries to come out of the low production phase to survive. The low production phase might be depression, recession or slowdown with the former being the worst and rare, government stake many new fiscal and monetary measures to boost demand and production and ultimately a recovery in an economy is managed. The business cycle of recovery may show the following major economy traits: (i) an upturn in aggregate (total) demand which has to be accompanied by increase in the level of production; (ii) production process expands and new investments become attractive; (iii) as demand goes upward, inflation also moves upward making borrowing cheaper for investors; (iv) with an upturn in production, new employment avenues are created and unemployment rate starts declining; etc.

www.insightsonindia.com 48 Insights IAS INSTA 75 Days REVISION PLAN for Prelims 2020 - InstaTests

48. The port integrity campaign has been launched by (a) International Maritime Organization (b) Maritime safety organizations (c) Maritime trade unions (d) Maritime Anti-Corruption Network

Solution: D

Port integrity campaign:

• Maritime Anti-Corruption Network starts port integrity campaign in India. • The campaign, which aims to reduce and (in the long term) eliminate integrity issues and bottlenecks to trade during operations in Indian ports, is a collective action of MACN, the Government of India, international organizations, and local industry stakeholders. • The main activities of the campaign include implementation of integrity training for port officials and the establishment of clear escalation and reporting processes. About MACN:

• Maritime Anti-Corruption Network (MACN) is a global business network of over 110 companies working to tackle corruption in the maritime industry. • Established in 2011 by a small group of committed maritime companies.

49. Consider the following statements regarding Minimum Support Price (MSP) for Minor Forest Produce (MFP) 1. MSP rate are decided by Ministry of Tribal Affairs. 2. The Scheme is applicable only in eight States having Schedule areas as listed in the Fifth Schedule. 3. It is a Centrally Sponsored Scheme. Which of the statements given above is/are correct? (a) 1 only (b) 2 and 3 only (c) 1 and 3 only (d) 1, 2 and 3

www.insightsonindia.com 49 Insights IAS INSTA 75 Days REVISION PLAN for Prelims 2020 - InstaTests

Solution: C

• Under the scheme “Mechanism for Marketing of Minor Forest Produce through Minimum Support Price and development of Value chain for MFP” Minimum Support Price (MSP) for Minor Forest Produce (MFP) has been fixed for select MFP. The scheme is designed as a social safety net for improvement of livelihood of MFP gatherers by providing them fair price for the MFPs they collect. Coverage

• The Scheme has been implemented in eight States having Schedule areas as listed in the Fifth Schedule of the constitution of India. From November 2016, the scheme is applicable in all States. What is this scheme all about?

• The Union Cabinet, in 2013, approved a Centrally Sponsored Scheme for marketing of non-nationalized / non monopolized Minor Forest Produce (MFP) and development of a value chain for MFP through Minimum Support Price (MSP). • This was a measure towards social safety for MFP gatherers, who are primarily members of the Scheduled Tribes (STs) most of them in Left Wing Extremism (LWE) areas. • The scheme had Rs. 967.28 crore as Central Government share and Rs. 249.50 crore as the States share for the current Plan period. • Key features of the scheme: Ensure that the tribal population gets a remunerative price for the produce they collect from the forest and provide alternative employment avenues to them. • Establish a system to ensure fair monetary returns for forest dweller’s efforts in collection, primary processing, storage, packaging, transportation etc, while ensuring sustainability of the resource base. • Get them a share of revenue from the sales proceeds with costs deducted. Coverage:

• Earlier, the scheme was extended only to Scheduled Areas in eight states and fixed MSPs for 12 MFPs. Later expanded to all states and UTs. Total number of MFPs covered under the list include more than 40 items. • Implementation: The responsibility of purchasing MFP on MSP will be with State designated agencies. • To ascertain market price, services of market correspondents would be availed by the designated agencies particularly for major markets trading in MFP. • The scheme supports primary value addition as well as provides for supply chain infrastructure like cold storage, warehouses etc.

www.insightsonindia.com 50 Insights IAS INSTA 75 Days REVISION PLAN for Prelims 2020 - InstaTests

• The Ministry of Tribal Affairs will be the nodal Ministry for implementation and monitoring of the scheme. The Minimum Support Price would be determined by the Ministry with technical help of TRIFED. • States have been given freedom for fixing MSP 10% above or below the MSP rate decided by Ministry of Tribal Affairs. Significance of the scheme:

• The Minor Forest Produce (MFP), also known as Non-Timber Forest Produce (NTFP), is a major source of livelihood and provides essential food, nutrition, medicinal needs and cash income to a large number of STs who live in and around forests. An estimated 100 million forest dwellers depend on the Minor Forest Produce for food, shelter, medicines, cash income, etc. • However, MFP production is highly dispersed spatially because of the poor accessibility of these areas and competitive market not having evolved. Consequently, MFP gatherers who are mostly poor are unable to bargain for fair prices. This package of intervention can help in organizing unstructured MFP markets.

50. Consider the following statements regarding Urban Haats 1. It has been launched by Ministry of Culture 2. It aims to provide direct marketing facilities to the craft persons and weavers. Which of the statements given above is/are correct? (a) 1 only (b) 2 only (c) Both 1 and 2 (d) Neither 1 nor 2

Solution: B

Urban Haats:

• It is an initiative of the Ministry of Textiles of the Government of India. • The objective of the scheme “Infrastructure and Technology Support” is to setup a permanent marketing infrastructure in big towns/ metropolitan cities to provide direct marketing facilities to the handicrafts artisans/handloom weavers. • The scheme is implemented through State Handicrafts/Handlooms Development Corporations/Tourism Development Corporations/ Urban Local Bodies with sufficient financial resources and organizational capacity to implement the project.

www.insightsonindia.com 51 Insights IAS INSTA 75 Days REVISION PLAN for Prelims 2020 - InstaTests

• The financial ceiling for Urban Haat is Rs. 300 lakh for each unit. 80% of the admissible amount is borne by the Office of the Development Commissioner (Handicrafts) and 20% contributed by the implementing agency.

DAY – 67 (InstaTest-67)

51. Consider the following statements regarding Annual study of state-level budgets released by RBI 1. Except during 2016-17, state governments have regularly met their fiscal deficit target of 3% of GDP 2. States now have a greater role to play in determining India’s GDP than the Centre. 3. Most states ended up meeting the fiscal deficit target not by increasing their revenues but by reducing their expenditure Which of the statements given above is/are correct? (a) 1, 2 and 3 (b) 2 only (c) 1 and 2 only (d) 3 only

Solution: A

Annual study of state-level budgets is released by RBI. Important Observations:

• Except during 2016-17, state governments have regularly met their fiscal deficit target of 3% of GDP • However, most states ended up meeting the fiscal deficit target not by increasing their revenues but by reducing their expenditure and increasingly borrowing from the market • States now have a greater role to play in determining India’s GDP than the Centre- States now spend one-and-a-half times more than the Union government • If a state, or all the states in aggregate, find it difficult to raise revenues, a rising mountain of debt — captured in the debt-to-GDP ratio — could start a vicious cycle.

www.insightsonindia.com 52 Insights IAS INSTA 75 Days REVISION PLAN for Prelims 2020 - InstaTests

Extra Reading The Fiscal Responsibility and Budget Management (FRBM) Act

• The Act was enacted in 2003 which set targets for the government to reduce fiscal deficits. The targets were put off several times. • Hence, in May 2016, the government set up a committee under NK Singh to review the FRBM Act. • The committee recommended that the government should target a fiscal deficit of 3% of the GDP in the years up to March 31, 2020, cut it to 2.8% in 2020-21 and to 2.5% by 2023. https://www.rbi.org.in/Scripts/AnnualPublications.aspx?head=State%20Finances%20:%20A %20Study%20of%20Budgets https://indianexpress.com/article/explained/rbi-state-budget-explained-gdp-economy- 6056708/

52. Consider the following statements regarding entrepreneurial intensity in India 1. A 10 per cent increase in registration of new firms per district-year yields an 18 per cent increase in GDDP (Gross Domestic District Product). 2. The peninsular states dominate entry of new firms and is restricted just to a few metropolitan cities. Which of the statements given above is/are correct? (a) 1 only (b) 2 only (c) Both 1 and 2 (d) Neither 1 nor 2

Solution: D

• Comparative entrepreneurial intensity: Between 2006 and 2016, the mean (median) number of new firms registered per year per 1000 workers was 0.10(0.11). In contrast, the mean (median) entrepreneurial intensity for the United Kingdom and the United States was 12.22 (11.84) and 12.12 (11.81) respectively. Dominance in Informal sector:

• India has low rates of entrepreneurship in the formal economy. In contrast to the other countries, a large number of India’s enterprises operate in the informal economy which is not captured in these data. www.insightsonindia.com 53 Insights IAS INSTA 75 Days REVISION PLAN for Prelims 2020 - InstaTests

• Entrepreneurship and GDP: The entrepreneurial activity is related to economic growth as it acts as an engine of economic growth and change in India. Regarding this survey observes the following: o A 10 per cent increase in registration of new firms per district-year yields a 1.8per cent increase in GDDP (Gross Domestic District Product) o Though the peninsular states dominate entry of new firms, entrepreneurship is dispersed across India and is not restricted just to a few metropolitan cities. o Spatial Heterogeneity exits in distribution of Entrepreneurial Activity in India i.e. varied level of entrepreneurial activity across the districts. The survey finds that all four regions except certain eastern states in India demonstrate strong growth in entrepreneurial activity over time. o While Gujarat’s labour reforms are viewed as pro-worker, the state has also passed other regulations that improve ease of doing business, including reduction in compliance burden, transparent and timely processing of approval and renewal of applications, etc. o Rajasthan too has introduced several reforms that are viewed as pro-employer. For example, to reduce the influence of trade unions, the state has increased the costs of union formation by increasing the minimum membership requirement to form a union to 30 per cent of the total workforce at an establishment, up from 15 per cent earlier.

53. Consider the following statements regarding SENSEX growth and pro- business sentiments 1. The share of services sector in the total number of companies on the Sensex has changed from being negligible in the 1980s to the dominant status today. 2. The difference between the sizes of the largest and smallest firms is rapidly increasing. Which of the statements given above is/are correct? (a) 1 only (b) 2 only (c) Both 1 and 2 (d) Neither 1 nor 2

Solution: A

www.insightsonindia.com 54 Insights IAS INSTA 75 Days REVISION PLAN for Prelims 2020 - InstaTests

Pro-Business, Creative destruction and Wealth creation Other than the SENSEX growth, pro- business sentiments also led to the following: o The years following 1991 liberalization saw the rapid emergence of new firms, new ideas, new technologies and new operating processes, causing a steep revision of the Sensex in 1996. o New sectors like banks and financials entered the index for the first time, eroding the predominance of the manufacturing sector on the index, placing the services sector on the map for the first time. o The diversity of sectors in the Sensex steadily increased over time following market reforms. The initial Sensex of 1986 was dominated by the Materials and Consumer Discretionary sectors, accounting for two-thirds of the firms on the Sensex. Sectors like financials, telecommunications and information technology were non-existent in the index then. With the entry of these new sectors, today’s Sensex is far less concentrated than the Sensex of the 1980sand 1990s, and mirrors the far lower sectoral concentration of the Indian economy as a whole. o Over the years, the share of services sector in the total number of companies on the Sensex has changed from being negligible in the 1980s to the dominant status today. The rising share in market capitalization has been accomplished by a rise in number of companies rather than a rise in size of incumbents, suggesting greater competition within these sectors. o The difference between the sizes of the largest and smallest firms is rapidly shrinking, and consequently monopoly power in the economy is declining and making way for more competitive markets. In 1991, the size of the largest firm was nearly 100 times the smallest firm which got reduced to 12 in 2018.

54. Consider the following statements regarding Information Fusion Centre (IFC) for the Indian Ocean Region (IOR) 1. It was established to gather intelligence on the movement of enemy submarines 2. It also facilitates the exchange of information on “white shipping” with countries in the region to improve maritime domain awareness in the Indian Ocean. Which of the statements given above is/are correct? (a) 1 only (b) 2 only (c) Both 1 and 2 (d) Neither 1 nor 2

Solution: B

www.insightsonindia.com 55 Insights IAS INSTA 75 Days REVISION PLAN for Prelims 2020 - InstaTests

About IFC- IOR:

• The IFC-IOR was established with the vision of strengthening maritime security in the region and beyond, by building a common coherent maritime situation picture and acting as a maritime information hub for the region. • The IFC has been established at the Navy’s Information Management and Analysis Centre (IMAC) in Gurugram, which is th e single point centre linking all the coastal radar chains to generate a seamless real-time picture of the nearly 7,500-km coastline. • Through this Centre, information on “white shipping”, or commercial shipping, will be exchanged with countries in the region to improve maritime domain awareness in the Indian Ocean. https://www.insightsonindia.com/2019/10/07/information-fusion-centre-ifc-for-the-indian- ocean-region-ior-4/

55. Consider the following statements regarding Economic freedom 1. The Index of Economic Freedom is brought out by the Heritage Foundation. 2. The Global Economic Freedom Index is brought out by the Fraser Institute. Which of the statements given above is/are correct? (a) 1 only (b) 2 only (c) Both 1 and 2 (d) Neither 1 nor 2

Solution: C

Economic freedom enhances wealth creation by enabling efficient allocation of entrepreneurial resources and energy to productive activities, thereby promoting economic dynamism. Facts: o The Index of Economic Freedom, which is brought out by the Heritage Foundation, India was categorized as ‘mostly unfree’ with a score of 55.2 in2019 ranking the Indian economy 129th among 186 countries, i.e., in the bottom 30 per cent of countries. o The Global Economic Freedom Index, which is brought out by the Fraser Institute, India ranks 79th among 162 countries with 108th rank in business regulation. The low rank in economic freedom makes it evident that India chains opportunities for wealth creation by shackling economic freedom for its citizens. www.insightsonindia.com 56 Insights IAS INSTA 75 Days REVISION PLAN for Prelims 2020 - InstaTests

56. Consider the following statements regarding Essential Commodities Act (ECA), 1955 1. It controls the production, supply and distribution of, and trade and commerce in, certain goods such as vegetables, pulses, edible oils, sugar etc., which are treated as essential commodities. 2. The powers to implement the provisions of the Act are delegated to the States. 3. ECA interferes in agricultural marketing by disincentivizing investments in warehousing and storage facilities. Which of the statements given above is/are correct? (a) 1 and 2 only (b) 2 and 3 only (c) 1 and 3 only (d) 1, 2 and 3

Solution: D

Essential Commodities Act (ECA), 1955 Features:

• Controls the production, supply and distribution of, and trade and commerce in, certain goods such as vegetables, pulses, edible oils, sugar etc., which are treated as essential commodities by Imposing Stockholding limit, restrict movement of goods, and mandate compulsory purchases under the system of levy. • The powers to implement the provisions of the Act are delegated to the States. • The purported aim of this Act is to ensure affordability of essential commodities for the poor by restricting hoarding.

• Impact: Creates Market Distortions: As agriculture is a seasonal activity, it is essential to store produce for the off-season to ensure smoothened availability of a product at stable prices throughout the year. • Therefore, producers face an inherent tradeoff between building an inventory in the harvest season and drawing down inventory in the lean season. ECA interferes with this mechanism by disincentivizing investments in warehousing and storage facilities due to frequent and unpredictable imposition of stock limits.

57. Consider the following statements regarding Green Crackers 1. The chemical formulation of green crackers produces water molecules, which substantially reduces emission levels and absorbs dust. 2. It promises a reduction in particulate matters and harmful gases, like nitrous oxide and sulfur oxide, by 90% per cent.

www.insightsonindia.com 57 Insights IAS INSTA 75 Days REVISION PLAN for Prelims 2020 - InstaTests

Which of the statements given above is/are correct? (a) 1 only (b) 2 only (c) Both 1 and 2 (d) Neither 1 nor 2

Solution: A

Green Crackers

• They are less harmful and less dangerous than the conventional ones. They are the crackers with reduced emission and decibel level. • They are known as ‘green’ firecrackers because they have a chemical formulation that produces water molecules, which substantially reduces emission levels and absorbs dust. • It promises a reduction in particulate matters and harmful gases, like nitrous oxide and sulfur oxide, by 30- 35 per cent and the green crackers will be 25-30 per cent cheaper to manufacture. • Indian Council of Scientific and Industrial Research (CSIR) has developed green crackers, which are new and improved formulations of the previous sound-emitting crackers and other fireworks

58. Consider the following statements regarding India’s Export scenario 1. Post the 1991 reforms, India’s share in merchandise (goods) exports has decreased at around 13 per cent per annum. 2. On an average, Network Products accounts for about 42 per cent of world manufactured exports. 3. Network Products exports accounts for a very small share (10 per cent in 2018) in India’s export basket. Which of the statements given above is/are correct? (a) 1 and 2 only (b) 2 and 3 only (c) 1 and 3 only (d) 1, 2 and 3

www.insightsonindia.com 58 Insights IAS INSTA 75 Days REVISION PLAN for Prelims 2020 - InstaTests

Solution: B

Facts on India’s Export scenario:

• Post the 1991 reforms, India’s share in merchandise (goods) exports has grown at 13.2 per cent per annum. • India’s share in world exports has increased from 0.6 per cent in 1991 to 1.7per cent in 2018. Yet, even by 2018, India’s world market share remains paltry compared to 12.8 per cent for China. • Imports of merchandise have grown faster (at the rate of 14.9 per cent per annum during 1993-2018) than exports, resulting in increasing trade deficits. World Exports of Network Products: Trends and Patterns

• World exports of NP increased steadily from US$ 2.01 Trillion in 2000 to US$5.41 Trillion in 2018. On an average, NP accounts for about 42 per cent of world manufactured exports. • Asia’s share in world exports of NP increased phenomenally from about 37 percent in 2000 to 51 per cent in 2018. East Asia accounted for the bulk of total Asian exports followed by Southeast. Rest of Asia (including South, Central and Western Asia) accounted for just 3 per cent of the total Asian exports. • India’s export of NP increased from about US$2 billion in 2000 to US$32 billion in 2018, its participation in this market remains miniscule compared to that of other Asian countries. Despite some increase, NP exports accounts for a very small share (10 per cent in 2018) in India’s export basket. In contrast, these products account for about one half of the total national exports of China, Japan and Korea. • Main category of NP exported by India is Road vehicles with a share of 4.9 percent in its total exports in 2018 (up from 1.3 per cent in 2000). In contrast, Electrical machinery, which accounts for the largest share in the export baskets of China (16.8 per cent) and Korea (30.5 per cent), accounts for less than 3 percent of India’s total exports.

59. Consider the following statements regarding Nationalization of banks 1. Public sector banks (PSBs) in India were formed through two waves of nationalizations, one in 1950 and the other in 1969. 2. After the nationalization, PSBs had a 91 per cent share in the national banking market. Which of the statements given above is/are correct? (a) 1 only (b) 2 only (c) Both 1 and 2 www.insightsonindia.com 59 Insights IAS INSTA 75 Days REVISION PLAN for Prelims 2020 - InstaTests

(d) Neither 1 nor 2

Solution: B

Banking structure: Since Nationalization till today

• The modern banking system in India has its roots in the colonial era starting in the 1800s. PSBs in India were formed through two waves of nationalizations, one in 1969 and the other in 1980. • After the 1980 nationalization, PSBs had a 91 per cent share in the national banking market with the remaining 9 per cent held by “old private banks” (OPBs) that were not nationalized. However, the share of PSBs has reduced to70% in recent times. Reduced stake has been absorbed by New Private Banks (NPBs) which came up in early 1990s after liberalization. The government exercises significant control over all aspects of PSB operations ranging from policies on recruitment and pay to investments and financing and bank governance including board and top management appointments. The majority ownership by the government also subjects PSB officers to scrutiny of their decisions by the central vigilance commission and the comptroller auditor general. With no real restrictions on what can be investigated and under what circumstances, officers of state- run banks are wary of taking risks in lending or in renegotiating bad debt, due to fears of harassment under the veil of vigilance investigations.

60. Mahamastakabhisheka, a great religious event, is associated with and done for whom of the following? (a) Nataraja (b) Buddha (c) Mahavir (d) Bahubali

Solution: D

• The Mahamastakabhisheka, refers to the abhiṣheka of the Jain images when held on a large scale. The most famous of such consecrations is the anointment of the Bahubali Gommateshwara Statue located at Shravanabelagola in Karnataka, India. It is an important Jain festival held once in every 12 years. (UPSC 2009 Prelims).

www.insightsonindia.com 60 Insights IAS INSTA 75 Days REVISION PLAN for Prelims 2020 - InstaTests

61. Consider the following statements regarding government receipts 1. Non tax revenue forms the largest share in government receipts. 2. In the tax revenues, GST and Corporation tax accounts for largest share. Which of the statements given above is/are correct? (a) 1 only (b) 2 only (c) Both 1 and 2 (d) Neither 1 nor 2

Solution: B

Receipts Highlights for 2020-21

• Total receipts (including borrowings) in 2020-21 are estimated to be Rs30,42,230 crore and net receipts (excluding borrowings) to be Rs 22,45,893 crore. Receipts (without borrowings) are estimated to increase by 16.3% over the revised estimates of 2019-20. • Gross tax revenue is budgeted to increase by 12% over the revised estimates of 2019-20, which is higher than the estimated nominal GDP growth of 10% in 2020-21. The net tax revenue of the central government(excluding states’ share in taxes) is estimated to be Rs 16,35,909 crore in2020-21.

www.insightsonindia.com 61 Insights IAS INSTA 75 Days REVISION PLAN for Prelims 2020 - InstaTests

• Devolution to states from center’s tax revenue is estimated to be Rs7,84,181 crore in 2020-21. In 2019-20, the devolution to states reduced by19% from an estimate of Rs 8,09,133 crore at the budgeted stage to Rs6,56,046 crore at the revised stage. • Non-tax revenue is expected to be Rs 3,85,017 crore in 2020-21. This is 11.4% higher than the revised estimate of 2019-20. • Capital receipts (without borrowings) are budgeted to increase by 175.7%over the revised estimates of 2019-20. This is on account of disinvestments, which are expected to be Rs 2,10,000 crore in 2020-21, as compared to Rs65,000 crore as per the revised estimates of 2019-20.

62. Consider the following statements regarding subsidies in India 1. Subsidies account for around 24% of the total expenditure by government. 2. The expenditure on major subsidies is around 1.4 per cent of GDP in 2019-20. 3. Of the total subsidies, Food subsidies account for the largest share. Which of the statements given above is/are correct? (a) 1 and 2 only (b) 2 and 3 only (c) 1 and 3 only (d) 1, 2 and 3

Solution: B

www.insightsonindia.com 62 Insights IAS INSTA 75 Days REVISION PLAN for Prelims 2020 - InstaTests

• The expenditure on major subsidies, which is a significant component of non committed revenue expenditure was pegged at 1.4 per cent of GDP in 2019-20 BE. The budgetary expenditure on major subsidies has shown a declining trend over the past years. Expenditure on Subsidies In 2020-21, the total expenditure on subsidies is estimated to be Rs 2,62,109 crore, a decrease of 0.5% from the revised estimate of 2019-20. This is largely due to a decrease in expenditure on fertilizer subsidy. Details are given below:

• Food subsidy: Allocation to food subsidy is estimated at Rs 1,15,570 crore in 2020- 21, a 6.3% increase as compared to the revised estimate of2019-20. In 2019-20 budget, Rs 1,84,220 crore was allocated to food subsidy. However, the revised estimate is much lower than the budgeted estimate at Rs 1,08,688 crore. This is due to a 41% cut (Rs 75,532 crore in amount) in the allocation to food subsidy for 2019-20 from the budgeted stage to the revised stage. • Fertilizer subsidy: Expenditure on fertilizer subsidy is estimated at Rs71,309 crore in 2020-21. This is a decrease of Rs 8,689 crore (10.9%) from the revised estimate of 2019-20. • Petroleum subsidy: Expenditure on petroleum subsidy is estimated to increase by 6.1% to Rs 40,915 crore in 2020-21. Petroleum subsidy consists of subsidy on LPG (Rs 37,256 crore) and kerosene subsidy (Rs 3,659 crore). In 2020-21, while the LPG subsidy is estimated to increase by Rs 3,170crore over the previous year, kerosene subsidy is estimated to decrease by Rs 824 crore. • Other subsidies: Expenditure on other subsidies includes interest subsidies for various government schemes, subsidies for the price support scheme for agricultural produce, and assistance to state agencies for procurement, among others. In 2020- 21, the expenditure on these other subsidies has decreased by Rs 1,987 crore (5.5%) over the revised estimate of 2019-20. Table 4 provides details of subsidies in 2020-21.

www.insightsonindia.com 63 Insights IAS INSTA 75 Days REVISION PLAN for Prelims 2020 - InstaTests

63. Consider the following statements regarding National e-Vidhan Application (NeVA) Project. 1. It is a Mission Mode Project (MMP) comes under the Digital India Programme. 2. The Ministry of Electronics and Information Technology (MietY) is Nodal Ministry’ for its implementation of the programme. 3. It aims to bring all the legislatures of the country together by technology. Which of the statements given above is/are correct? (a) 1 only (b) 2 and 3 only (c) 1 and 3 only (d) 1, 2 and 3

Solution: C

National e-Vidhan Application (NeVA) Project:

• Minister of State for Parliamentary Affairs and Water Resources, River Development and Ganga Rejuvenation, gave details on the status of implementation of National e- Vidhan Application (NeVA) Project. e-Vidhan:

• It is a Mission Mode Project (MMP) comes under the Digital India Programme. • Ministry of Parliamentary Affairs (MoPA) is the ‘Nodal Ministry’ for its implementation in all the 31 States/UTs with Legislatures. • The funding for e-Vidhan is provided by the MoPA and technical support by Ministry of Electronics and Information Technology (MietY). • The funding of NeVA is through Central Sponsored Scheme i.e. 60:40; and 90:10 for North East & hilly States and 100% for UTs. Aim of NeVA:

• It aims to bring all the legislatures of the country together, in one platform thereby creating a massive data depository without having the complexity of multiple applications. • Paperless Assembly or e-Assembly is a concept involving of electronic means to facilitate the work of Assembly. • It enables automation of entire law making process, tracking of decisions and documents, sharing of information. • Through the cloud technology (Meghraj), data deployed can be accessed anywhere at any time.

www.insightsonindia.com 64 Insights IAS INSTA 75 Days REVISION PLAN for Prelims 2020 - InstaTests

• The live webcasting of Lok Sabha TV and Rajya Sabha TV is also available on this application. • Doordarshan has already been enabled with provision to incorporate similar facility in respect of State Legislatures. • Himachal Pradesh is already the first Digital Legislature of the country.

64. Consider the following statements regarding import basket 1. China continues to be the largest exporter to India followed by USA, UAE and Saudi Arabia. 2. In the import basket of 2019-20, petroleum products had the largest share. 3. Between 2011-12 and 2019-20, imports of Electronics grew the fastest from a negligible share to 3.6 per cent of import basket. Which of the statements given above is/are correct? (a) 1 and 2 only (b) 2 and 3 only (c) 1 and 3 only (d) 1, 2 and 3

Solution: C

www.insightsonindia.com 65 Insights IAS INSTA 75 Days REVISION PLAN for Prelims 2020 - InstaTests

• In the import basket of 2019-20 (April-November), crude petroleum had the largest share followed by gold and petroleum products. However, between2011-12 and 2019- 20, imports of Electronics grew the fastest from a negligible share to 3.6 per cent (Figure 15).

• China continues to be the largest exporter to India followed by USA, UAE and Saudi Arabia. In recent times, Hong Kong, Korea and Singapore have also emerged as significant exporters to India (Figure 16).

65. With which of the following countries does India have trade deficit? 1. China 2. Saudi Arabia 3. UAE 4. Germany Select the correct answer using the code given below: (a) 1, 2 and 3 only (b) 2, 3 and 4 only (c) 1, 2 and 4 only (d) 1, 2, 3 and 4

Solution: C

www.insightsonindia.com 66 Insights IAS INSTA 75 Days REVISION PLAN for Prelims 2020 - InstaTests

• The bilateral trade position with respect to top trading partners over a period of time is shown in Table 3. With two top trading countries i.e. USA and United Arab Emirates, India has consistently run trade surplus since 201415. On the other hand, India has trade deficit continuously since 2014-15with respect to other major trading partners i.e. China PRP, Saudi Arabia, Iraq, Germany, Korea RP, Indonesia and Switzerland. India had trade surplus with Hong Kong and Singapore till 2017-18, before it changed to trade deficit in 2018-19. The bilateral imbalances have remained stable in most cases.

66. Consider the following statements regarding Institutions of Eminence Scheme 1. This has been launched by University Grants Commission (UGC). 2. Under the scheme, there will be financial assistance to the private institutions based on their performance. 3. Under the scheme, selected institutions will have greater autonomy compared to other higher education institutions. Which of the statements given above is/are correct? (a) 1 only (b) 2 and 3 only (c) 1 and 3 only (d) 1, 2 and 3

Solution: C www.insightsonindia.com 67 Insights IAS INSTA 75 Days REVISION PLAN for Prelims 2020 - InstaTests

Institutions of Eminence scheme:

• The scheme of IoE was rolled out by University Grants Commission (UGC). • It aims to project Indian institutes to global recognition. • The selected institutes will enjoy complete academic and administrative autonomy. • The selection shall be made through challenge method mode by the Empowered Expert Committee constituted for the purpose. • Eligibility: Only higher education institutions currently placed in the top 500 of global rankings or top 50 of the National Institutional Ranking Framework (NIRF) are eligible to apply for the eminence tag. • The private Institutions of Eminence can also come up as Greenfield ventures- provided the sponsoring organisation submits a convincing perspective plan for 15 years. • There will be no financial assistance to the private institutions.

67. Consider the following statements regarding Net Services 1. An increase in service exports to GDP ratio has a net positive impact on the BoP position. 2. Over the years, service imports in relation to GDP has been steadily rising. 3. India’s net services surplus has been steadily increasing in relation to GDP. Which of the statements given above is/are correct? (a) 1 and 2 only (b) 2 and 3 only (c) 1 and 3 only (d) 1, 2 and 3

Solution: A

Net Services: India’s net services surplus has been steadily declining in relation to GDP since it reached its peak to about two-thirds of merchandise deficit in 2016-17. Services Exports: An increase in service exports to GDP ratio has a net positive impact on the BoP position.

• India’s service exports have consistently hovered between 7.4 to 7.7per cent of GDP. • Software services constitute around 40-45 per cent, followed by business services at about 18-20 per cent, travel at 11-14 per cent and transportation at 9-11 per cent of service exports.

www.insightsonindia.com 68 Insights IAS INSTA 75 Days REVISION PLAN for Prelims 2020 - InstaTests

Service Imports: An increase in service imports to GDP ratio has a net negative impact on the BoP position.

• Over the years, service imports in relation to GDP has been steadily rising given arising level of FDI and a gradual up scaling of the Make in India program. • Business services constitute about a third of service imports and the component of travel services has been steadily increasing reflecting the growing attractiveness of global destinations to the domestic tourists in the country.

68. Consider the following statements regarding Net Remittances 1. The Migration Report 2019 released by the United Nations has placed India as the leading country of origin of international migrants in 2019. 2. According to a report of World Bank, India remained the top remittance recipient country in 2018 followed by China. Which of the statements given above is/are correct? (a) 1 only (b) 2 only (c) Both 1 and 2 (d) Neither 1 nor 2

Solution: C

Net Remittances: An increase in net remittances improves the BoP position.

• Net remittances from Indians employed overseas has increased with the amount received in the first half of 2019-20 being more than 50per cent of the total receivables in 2018-19. • The Migration Report 2019 released by the United Nations has placed India as the leading country of origin of international migrants in 2019 with a Diaspora strength of 17.5 million. • Further, as per the October 2019 report of World Bank, India remained the top remittance recipient country in 2018 followed by China.

69. Consider the following statements regarding Price stabilization fund 1. It has been launched under APMC Act 2. It is Central Sector Scheme 3. It aims to support market interventions for price control of perishable agri- horticultural commodities. www.insightsonindia.com 69 Insights IAS INSTA 75 Days REVISION PLAN for Prelims 2020 - InstaTests

Which of the statements given above is/are correct? (a) 1 and 2 only (b) 2 and 3 only (c) 1 and 3 only (d) 1, 2 and 3

Solution: B

Price stabilization fund:

• It is Central Sector Scheme • Price Stabilization Fund (PSF) refers to any fund constituted for the purpose of containing extreme volatility in prices of selected commodities • The amount in the fund is generally utilized for activities aimed at bringing down/up the high/low prices say for instance, procurement of such products and distribution of the same as and when required, so that prices remain in a range. • Many countries use such dedicated funds for stabilization of major petroleum product prices, particularly if they are importers. • Some countries use such funds for stabilizing not just commodity prices but a variety of key macroeconomic variables such as the exchange rate (which is nothing but the price of the domestic currency expressed in terms of an external currency), benchmark stock indices etc. • It aims to support market interventions for price control of perishable agri- horticultural commodities.

70. Which of the following is/are the reasons for downward trend in yield on G-Sec? 1. Change in monetary policy stance of the U.S. Fed. 2. Easing of liquidity condition of the banking system. 3. Transfer of RBI surplus to the Government. 4. Significant and sustained surplus liquidity. Select the correct answer using the code given below: (a) 1, 2 and 3 only (b) 2, 3 and 4 only (c) 1, 3 and 4 only (d) 1, 2, 3 and 4

www.insightsonindia.com 70 Insights IAS INSTA 75 Days REVISION PLAN for Prelims 2020 - InstaTests

Solution: D

Developments in G-sec Market: Initially yield on G-Sec hardened marginally on account of rise in crude oil prices. Thereafter, it largely followed a downward trend which may be attributed to various reasons:

• Change in monetary policy stance of the U.S. Fed (on global growth concerns and ongoing trade tensions). • Easing of liquidity condition of the banking system. • Consecutive policy rate cuts by the RBI along with change of stance from neutral to accommodative. • Benign crude oil prices aided the sentiment. • Transfer of RBI surplus to the Government. • Significant and sustained surplus liquidity. • Special Open Market Operation (purchase of long-term securities and simultaneous sale of short-term securities) by Reserve Bank of India also helped bring down the yield slightly on 10-year G-Secs.

71. Consider the following statements regarding PRAGATI platform. 1. It has been launched by Department of Administrative Reforms and Public Grievances 2. It is multi-modal platform for Pro-Active Governance and Timely Implementation of programmes and projects Which of the statements given above is/are correct? (a) 1 only (b) 2 only (c) Both 1 and 2 (d) Neither 1 nor 2

Solution: B

PRAGATI (Pro-Active Governance and Timely Implementation):

• The Prime Minister recently chaired his thirtieth interaction through PRAGATI – the ICT-based, multi-modal platform for Pro-Active Governance and Timely Implementation.

www.insightsonindia.com 71 Insights IAS INSTA 75 Days REVISION PLAN for Prelims 2020 - InstaTests

• PRAGATI (Pro-Active Governance and Timely Implementation) is a unique integrating and interactive platform. • The platform is aimed at addressing common man’s grievances, and simultaneously monitoring and reviewing important programmes and projects of the Government of India as well as projects flagged by State Governments. • The PRAGATI platform uniquely bundles three latest technologies: Digital data management, video-conferencing and geo-spatial technology. • It also offers a unique combination in the direction of cooperative federalism since it brings on one stage the Secretaries of Government of India and the Chief Secretaries of the States. • With this, the Prime Minister is able to discuss the issues with the concerned Central and State officials with full information and latest visuals of the ground level situation. It is also an innovative project in e-governance and good governance. • It is a three-tier system (PMO, Union Government Secretaries, and Chief Secretaries of the States). • Issues to be flagged before the PM are picked up from the available database regarding Public Grievances, on-going Programmes and pending Projects.

72. Consider the following statements regarding India’s progress towards SDGs 1. NITI Aayog has come up with a single measurable SDG index to track the progress of all the States and UTs. 2. The SDG score varies from 0 to 100. A score of 100 implies that the States/UTs have achieved the targets set for 2030. 3. India’s growth trajectory for achieving SDG 10 (Reduced Inequality) and SDG 15 (Life on Land) is impressive as compared to the other SDGs. Which of the statements given above is/are correct? (a) 1 and 2 only (b) 2 and 3 only (c) 1 and 3 only (d) 1, 2 and 3

Solution: D

India’s progress towards SDGs

• NITI Aayog has come up with a single measurable SDG index to track the progress of all the States and UTs across 13 out of 17 SDGs (excluding Goal 12, 13, 14 and 17 on

www.insightsonindia.com 72 Insights IAS INSTA 75 Days REVISION PLAN for Prelims 2020 - InstaTests

account of unavailability of comparable data across States/UTs), it helps in informed policy formulations. • The SDG score varies from 0 to 100. A score of 100 implies that the States/UTs have achieved the targets set for 2030; a score of 0implies that the particular State/UT is at the bottom of the table. • States with scores equal to/greater than 65 are considered as Front-Runners; in the range of 50-64 as Performers and as Aspirants if the score is less than 50. Kerala and Himachal Pradesh are the frontrunners amongst all the states. It is noteworthy that none of the States/ UTs fall in the Aspirant category in 2019. • Success – India’s growth trajectory for achieving SDG 10 (Reduced Inequality) and SDG 15 (Life on Land) is impressive as compared to the other SDGs as several states have achieved 100 in these SDGs. This may be due to performance in worthy initiatives such as PMJDY, Mahatma Gandhi National Rural Employment Guarantee Act (MGNREGA), the National Environment Policy, National Agro-forestry Policy and Green Highways Policy. • Struggling – India is struggling to achieve its targets of SDG 5 (Gender Equality) and SDG. 11 (Sustainable Cities and Communities) as large number of states are in the ‘Aspirants’ category’. Goa, a front runner among all the States and UTs in SDG 11, has been doing exceptionally well in waste management.

73. The Kandangi saree recently got GI tag, belongs to the state of: (a) Karnataka (b) Tamil Nadu (c) Maharashtra (d) Andhra Pradesh

Solution: B

GI Atal for Dindigul lock and Kandangi saree: Two products from Tamil Nadu — Dindigul lock and Kandangi Saree — have been given the Geographical Indication (GI) tag by The Geographical Indications Registry in Chennai. 1. The Dindigul locks are known throughout the world for their superior quality and durability, so much so that even the city is called Lock City. 2. The Kandangi sarees are manufactured in the entire Karaikudi taluk in Sivaganga district. They are characterized by large contrast borders and some are known to have borders covering as far as two-thirds of the saree which is usually around 5.10 m-5.60 m in length.

www.insightsonindia.com 73 Insights IAS INSTA 75 Days REVISION PLAN for Prelims 2020 - InstaTests

74. Consider the following statements regarding Food processing sector 1. The sector constituted as much as 8.83 per cent and 10.66 per cent of GVA in Manufacturing and Agriculture sector respectively. 2. The value of processed food exports during 2018-19 accounts for about 10 per cent of India’s total exports. 3. The sector has been growing at an average annual growth rate of around 15 per cent. Which of the statements given above is/are correct? (a) 1 and 2 only (b) 2 and 3 only (c) 1 and 3 only (d) 1, 2 and 3

Solution: A

Food processing sector

• A well-developed food processing sector with higher level of processing helps in the reduction of wastage, improves value addition, promotes crop diversification, ensures better return to the farmers, promotes employment as well as increases export earnings. Growth in the food processing sector is also expected to open up opportunities for players having strong linkages in the agri-value chain. During the last 6 years ending 2017-18, Food Processing Industries sector has been growing at an average annual growth rate of around 5.06 per cent. The sector constituted as much as 8.83 per cent and 10.66 per cent of GVA in Manufacturing and Agriculture sector respectively in 2017-18 at 2011-12 prices. 7.32 According to the Annual Survey of Industries for 2016-17, the total number of persons engaged in registered food processing sector was 18.54 lakhs. 7.33 Unregistered food processing sector supports employment to 51.11 lakh workers as per the NSSO 73rd Round, 2015-16 and constitutes 14.18 per cent of employment in the unregistered manufacturing sector. The value of processed food exports during 2018-19 was of the order of US $ 35.30 billion accounting for about 10.70 per cent of India’s total exports (total exports US $ 330.08 billion). The value of import of processed food during 2018-19 was US $ 19.32 billion which is 3.76 per cent of India’s total imports.

75. Advance Pricing Agreements (APAs), sometime seen in the news, is related to (a) Taxation (b) Allocation of spectrum and charges on consumer www.insightsonindia.com 74 Insights IAS INSTA 75 Days REVISION PLAN for Prelims 2020 - InstaTests

(c) Mine leasing agreement (d) Solar energy purchase by DISCOMS

Solution: A

• An advance pricing agreement (APA) is an ahead-of-time agreement between a taxpayer and a tax authority on an appropriate transfer pricing methodology (TPM) for a set of transactions at issue over a fixed period of time (called “Covered Transactions”). https://economictimes.indiatimes.com/news/economy/finance/cbdt-inks-300th-advance- pricing-agreement/articleshow/71395453.cms?from=mdr

DAY – 68 (InstaTest-68)

76. Consider the following statements regarding Satnami sect 1. Satnami sect was founded by a saint named “Birbhan” in 1657 in Narnaul in Haryana. 2. The followers of this sect kept their heads shaven and abstained from liquor and meat. Which of the statements given above is/are correct? (a) 1 only (b) 2 only (c) Both 1 and 2 (d) Neither 1 nor 2

Solution: C

• The Satnami sect was founded by a saint named “Birbhan” in 1657 in Narnaul in Haryana. The major religious activity of this sect is to chant and meditate the true names {Sat-Nam} of God specially Rama and . • The followers of this sect kept their heads shaven (thus called Mundiyas) and abstained from liquor and meat.

www.insightsonindia.com 75 Insights IAS INSTA 75 Days REVISION PLAN for Prelims 2020 - InstaTests

Satnami Rebellion

• The revolt triggered when a Mughal soldier killed a Satnami. The Satnamis killed the soldier in revenge and in turn Mughal soldiers were sent to teach them a lesson. Some 5,000 Satnamis stood up in arms and routed the Mughal troops in the town, drove away the Mughal administrators and set up their own administration under their leader Birbhan.

77. Consider the following statements regarding emission standards in India 1. Vehicle emission norms were introduced in India in 1991 for petrol and in 1992 for diesel vehicles. 2. Euro norms are followed in India under the name Bharat Stage Emission Standards. 3. The standards and the timeline for implementation are set by the Central Pollution Control Board. Which of the statements given above is/are correct? (a) 1 and 2 only (b) 2 and 3 only (c) 1 and 3 only (d) 1, 2 and 3

Solution: D

Definition of ‘Bharat Emission Standards’ Definition: These are the standards set up by the Indian government which specify the amount of air pollutants from internal combustion engines, including those that vehicles can emit. If these emit more pollutants than the prescribed limit, they don’t get a clearance to be sold in an open market. Bharat Stage Emission Standards have been instituted by the Central Pollution Control Board (CPCB), instituted within the Ministry of Environment Forests and Climate Change. Description: Vehicle emission norms were introduced in India in 1991 for petrol and in 1992 for diesel vehicles. Since 2000, Euro norms are followed in India under the name Bharat Stage Emission Standards for four wheeled vehicles. Bharat stage III norms have been enforced across India since October 2010. In a few cities, Bharat stage IV norms are in place since April 2010. Bharat stage IV is proposed to be enforced throughout India by April 2017. It has already been put into use in 13 major cities.

www.insightsonindia.com 76 Insights IAS INSTA 75 Days REVISION PLAN for Prelims 2020 - InstaTests

Upgrading the emission norms requires the manufacturing companies to upgrade their technology, which in turn increases the cost of the vehicle. Cost is one of the main reasons for the slow upgrade of emission standards. However, there are also arguments that the increase in cost is made up by savings in health costs as the pollutants causing diseases are decreased with the upgrade in emission standards. Fuels also play a crucial role in meeting these emission norms. Fuel specifications have also been aligned to its corresponding European production norms.

• It is to regulate the output of air pollutants from internal combustion engines and Spark-ignition engines equipment, including motor vehicles. • It includes both emission standards for new vehicles as well as specifications for commercial petrol and diesel fuels. • The standards and the timeline for implementation are set by the Central Pollution Control Board under the Ministry of Environment & Forests and climate change. • All new vehicles manufactured after the implementation of BS norms must be compliant with the regulations. • Major emissions governed under these norms are carbon monoxide emissions, hydrocarbon emission limits. Nitrogen Oxides and particulate matter are also significant metrics. • BS-IV standard was brought into place in country-wide in April 2017.

78. Consider the following statements regarding Taj Trapezium Zone (TTZ) 1. It is a defined area of 10,400 sq km around the Taj Mahal to protect the monument from pollution. 2. The TTZ comprises only two World Heritage Sites the Taj Mahal and Agra Fort. Which of the statements given above is/are correct? (a) 1 only (b) 2 only (c) Both 1 and 2 (d) Neither 1 nor 2

Solution: A

• Taj Trapezium Zone (TTZ) is a defined area of 10,400 sq km around the Taj Mahal to protect the monument from pollution. The Supreme Court of India delivered a ruling on December 30, 1996 regarding industries covered under the TTZ, in response to a PIL seeking to protect the Taj Mahal from environmental pollution. It banned the use of coal/ coke in industries located in the TTZ with a mandate for switching over from

www.insightsonindia.com 77 Insights IAS INSTA 75 Days REVISION PLAN for Prelims 2020 - InstaTests

coal/ coke to natural gas, and relocating them outside the TTZ or shutting down. The TTZ comprises monuments including three World Heritage Sites the Taj Mahal, Agra Fort and Fatehpur Sikri. TTZ is so named since it is located around the Taj Mahal and is shaped like a trapezoid. • By the order of Ministry of Environment and Forests in exercise of the powers conferred by the Environment Protection Act, 1986 and in supersession of the order of the government of India in the ministry of Environment and forests, number S.O. 350(E), dated 17th May, 1999, except as thins done or omitted to be done before such supersession, the central Government hereby re-constitutes the Taj Trapezium Zone Pollution (Prevention and Control) authority. Area Under TTZ

• The geographical limits of the Taj Trapezium Zone is defined in the shape of trapezoid between 26° 45N & 77 ° 15E to 27 ° 45N & 77° 15E in the West of the Taj Mahal and in the East of Taj Mahal between 27° 00N & 78° 30E to 27° 30E, lying in the Agra Division of the State of Uttar Pradesh and in the Bharatpur Division of the State of Rajasthan. Responsibility areas :

• Protecting Taj Mahal from damage via pollution • Control Pollution • Keep record of Taj Mahal pollution statics • Study city pollution

79. Global competitiveness index, sometime seen in the news, is published by (a) OECD (b) World Bank (c) AIIB (d) World Economic Forum

Solution: D

• Global Competitiveness Index released by World Economic Forum • The Global Competitiveness Index (GCI), which was launched in 1979, maps the competitiveness landscape of 141 economies through 103 indicators organised into 12 pillars.

www.insightsonindia.com 78 Insights IAS INSTA 75 Days REVISION PLAN for Prelims 2020 - InstaTests

• Compared to last year, India has moved down 10 places to rank 68th. India was ranked 58th last year. It is among the worst-performing BRICS nations along with Brazil, ranked even lower than India at 71st this year.

80. Consider the following statements regarding Categories of biofuels 1. First-generation biofuels are made from sugar, starch, vegetable oil, or animal fats using conventional technology. 2. Second generation biofuels are produced from non-food crops, such as cellulosic biofuels and waste biomass. 3. Third generation biofuels are aimed at producing sustainable energy and also capturing and storing carbon dioxide. Which of the statements given above is/are correct? (a) 1 and 2 only (b) 2 only (c) 1 and 3 only (d) 1, 2 and 3

Solution: A

Biofuels are liquid or gaseous fuels primarily produced from biomass, and can be used to replace or can be used in addition to diesel, petrol or other fossil fuels for transport, stationary, portable and other applications. Crops used to make biofuels are generally either high in sugar (such as sugarcane, sugar beet, and sweet sorghum), starch (such as maize and tapioca) or oils (such as soybean, rapeseed, coconut, sunflower). Categories of biofuels Biofuels are generally classified into three categories. They are 1. First generation biofuels – First-generation biofuels are made from sugar, starch, vegetable oil, or animal fats using conventional technology. Common first-generation biofuels include Bioalcohols, Biodiesel, Vegetable oil, Bioethers, Biogas. 2. Second generation biofuels – These are produced from non-food crops, such as cellulosic biofuels and waste biomass (stalks of wheat and corn, and wood). Examples include advanced biofuels like biohydrogen, biomethanol. 3. Third generation biofuels – These are produced from micro-organisms like algae. Fourth Generation Bio fuels

• Aimed at producing sustainable energy and also capturing and storing carbon dioxide. • Carbon dioxide is captured which can be then geo-sequestered. www.insightsonindia.com 79 Insights IAS INSTA 75 Days REVISION PLAN for Prelims 2020 - InstaTests

• This is carbon neutral technology.

81. Consider the following statements regarding Gandhi Solar Park 1. It has been setup at Sabarmati Ashram. 2. The solar panels are powered up to reach the max of 50 KW of generation power. 3. As a symbolic effort, one panel at the park is dedicated each for every 193 UN Member State. Which of the statements given above is/are correct? (a) 1 and 2 only (b) 2 and 3 only (c) 3 only (d) 1, 2 and 3

Solution: B

• In a first of its kind symbolic effort by India at the UN, Prime Minister Narendra Modi will inaugurate a 50KW ‘Gandhi Solar Park’ next week during his visit to the world organization, a gesture that highlights India’s willingness to go beyond the talk on climate change. • At a contribution of about one million dollars, India has gifted solar panels that have been installed on the roof of the UN Headquarters here, one panel each for every 193 UN Member State. • Modi will remotely inaugurate the solar park at the UN Headquarters and the ‘Gandhi Peace Garden’ during a special commemorative event on September 24 marking Gandhi’s 150th birth anniversary. On the occasion, a special UN Postage on Gandhi’s 150 years will also be released. • The ‘Gandhi Peace Garden’ is an innovative initiative under which the Consulate General of India in New York, Long Island-based NGO Shanti Fund and the State University of New York – Old Westbury have entered into an agreement to plant 150 trees. The garden is dedicated in the memory of Gandhi and is a crowdsourced project, with people adopting trees in the memory of their loved ones. The garden is in an open site within the 600 acres campus of the University. • The solar panels are powered up to reach the max of 50 KW of generation power.

www.insightsonindia.com 80 Insights IAS INSTA 75 Days REVISION PLAN for Prelims 2020 - InstaTests

82. Consider the following statements regarding Mahatma Gandhi National Fellowship Programme 1. It aims to address the challenge of non-availability of personnel for implementation of various programmes at national, state and district levels 2. It aims to increase the number of personnel available for MNREGA works 3. It is designed under the Ministry of Rural Development Which of the statements given above is/are correct? (a) 1 and 2 only (b) 1 and 3 only (c) 2 only (d) 1 only

Solution: D

Mahatma Gandhi National Fellowship Programme

• Ministry of Skill Development and Entrepreneurship launches Mahatma Gandhi National Fellowship Programme with IIM Bangalore. It is a two-year fellowship programme to boost skill development at the district level. • Designed under Skills Acquisition and Knowledge Awareness for Livelihood Promotion (SANKALP). It aims to address the challenge of non-availability of personnel for implementation of various programmes at national, state and district levels • Eligibility: Eligible fellows for the programme have to be in 21-30 years age-group, have a graduation degree from a recognized university and be citizens of India. Proficiency in official language of state of fieldwork will be mandatory. • Fellows will receive a stipend of Rs. 50,000 in the first year and Rs. 60,000 in the second year

83. Consider the following statements regarding Bamboonomics 1. It was launched by Ministry of Environment, Forest and Climate Change. 2. It is one of the tribal movement, to promote tribal enterprise. 3. It was launched for combating desertification and the climate change at COP 14 to the UNCCD. Which of the statements given above is/are correct? (a) 2 only (b) 2 and 3 only

www.insightsonindia.com 81 Insights IAS INSTA 75 Days REVISION PLAN for Prelims 2020 - InstaTests

(c) 1 and 3 only (d) 1, 2 and 3

Solution: B

Bamboonomics

• Ministry of Tribal Affairs launched the biggest Tribal movement, to promote tribal enterprise through ‘Bamboonomics’. • It was launched for combating desertification and the climate change at COP 14 to the UNCCD. • TRIFED introduced the ‘The 4P1000 Initiative: The Tribal Perspective through Bamboonomics’. • The international initiative “4 per 1000” was launched by France in 2015 at the COP 21. The aim of the initiative is to,

• Demonstrate that agriculture, and in particular agricultural soils can play a crucial role where food security and climate change are concerned. • Encourage stakeholders to transition towards a productive, highly resilient agriculture, based on the appropriate management of lands and soils, creating jobs and incomes hence ensuring sustainable development. • An annual growth rate of 0.4% in the soil carbon stocks, or 4‰ per year, in the first 30-40 cm of soil, would significantly reduce the CO2 concentration in the atmosphere related to human activities. • This is what the 4 per 1000 Initiative proposes, soils for food security and climate. • The initiative is intended to complement those necessary efforts to reduce greenhouse gas emissions globally. • It is voluntary, it is up to each member to define how they want to contribute to the goals.

84. Consider the following statements regarding International Seed Treaty 1. It was adopted by Conference of the Food and Agriculture Organization of the United Nations. 2. It aims at guaranteeing food security through the conservation, exchange and sustainable use of the world’s plant genetic resources for food and agriculture (PGRFA). Which of the statements given above is/are correct? (a) 1 only

www.insightsonindia.com 82 Insights IAS INSTA 75 Days REVISION PLAN for Prelims 2020 - InstaTests

(b) 2 only (c) Both 1 and 2 (d) Neither 1 nor 2

Solution: C

The International Treaty on Plant Genetic Resources for Food and Agriculture was adopted by the Thirty-First Session of the Conference of the Food and Agriculture Organization of the United Nations on 3 November 2001. The Treaty aims at:

• recognizing the enormous contribution of farmers to the diversity of crops that feed the world; • establishing a global system to provide farmers, plant breeders and scientists with access to plant genetic materials; • ensuring that recipients share benefits they derive from the use of these genetic materials with the countries where they have been originated. Main Provisions: Multilateral system

• The Treaty’s truly innovative solution to access and benefit sharing, the Multilateral System, puts 64 of our most important crops – crops that together account for 80 percent of the food we derive from plants – into an easily accessible global pool of genetic resources that is freely available to potential users in the Treaty’s ratifying nations for some uses. Access and benefit sharing

• The Treaty facilitates access to the genetic materials of the 64 crops in the Multilateral System for research, breeding and training for food and agriculture. Those who access the materials must be from the Treaty’s ratifying nations and they must agree to use the materials totally for research, breeding and training for food and agriculture. The Treaty prevents the recipients of genetic resources from claiming intellectual property rights over those resources in the form in which they received them, and ensures that access to genetic resources already protected by international property rights is consistent with international and national laws. • Those who access genetic materials through the Multilateral System agree to share any benefits from their use through four benefit-sharing mechanisms established by the Treaty.

www.insightsonindia.com 83 Insights IAS INSTA 75 Days REVISION PLAN for Prelims 2020 - InstaTests

International Seed Treaty

• International Treaty of Plant Genetic Resources for Food and Agriculture (ITPGRFA), is popularly known as the International Seed Treaty. • It is a comprehensive international agreement, which aims at guaranteeing food security through the conservation, exchange and sustainable use of the world’s plant genetic resources for food and agriculture (PGRFA). It also recognizes farmers’ rights, subject to national laws to:

• The protection of traditional knowledge relevant to plant genetic resources for food and agriculture; • The right to equitably participate in sharing benefits arising from the utilization of plant genetic resources for food and agriculture. • The right to participate in making decisions, at the national level, on matters related to the conservation and sustainable use of plant genetic resources for food and agriculture. • The Treaty establishes the Multilateral System of Access and Benefit-sharing to facilitate plant germplasm exchanges and benefit sharing through Standard Material Transfer Agreement (SMTA). • The treaty was negotiated by the Food and Agriculture Organization of the United Nations (FAO) Commission on Genetic Resources for Food and Agriculture (CGRFA) and since 2006 has its own Governing Body under the aegis of the FAO. • The Governing Body is the highest organ of the Treaty and holds biennial meetings.

85. A new type of El Nino called El Nino Modoki appeared in the news. In this context, consider the following statements: 1. Normal El Nino forms in the Central Pacific ocean whereas El Nino Modoki forms in Eastern Pacific ocean. 2. Normal El Nino results in diminished hurricanes in the Atlantic Ocean but El Nino Modoki results in a greater number of hurricanes with greater frequency. Which of the statements given above is/are correct? (a) 1 only (b) 2only (c) Both 1 and 2 (d) Neither I nor 2

Solution: B

www.insightsonindia.com 84 Insights IAS INSTA 75 Days REVISION PLAN for Prelims 2020 - InstaTests

• El Niño Modoki is a coupled ocean-atmosphere phenomenon in the tropical Pacific. It is different from another coupled phenomenon in the tropical Pacific namely, El Niño. • Conventional El Niño is characterized by strong anomalous warming in the eastern equatorial Pacific. Whereas, El Niño Modoki is associated with strong anomalous warming in the central tropical Pacific and cooling in the eastern and western tropical Pacific • Normal El Nino results in diminished hurricanes in the Atlantic Ocean but El Nino Modoki results in a greater number of hurricanes with greater frequency. (UPSC 2010)

86. Consider the following statements regarding Mission 175 1. Mission 175 is a global platform to explore strategies for development and deployment of renewables. 2. It is hosted by the Ministry of New and Renewable Energy (MNRE). Which of the statements given above is/are correct? (a) 1 only (b) 2 only (c) Both 1 and 2 (d) Neither 1 nor 2

Solution: C

Mission 175

• Mission 175 is a global platform to explore strategies for development and deployment of renewables. • It is hosted by the Ministry of New and Renewable Energy (MNRE), the event showcases India’s renewable energy potential and the Government’s efforts to scale up capacity to meet the national energy requirement in a socially, economically and ecologically sustainable manner. • In the year 2015, the Government of India announced a target for 175 GW cumulative renewable power installed capacity by the year 2022. • Ahead of COP 21, India submitted its Intended Nationally Determined Contribution (INDC) to the UNFCCC, outlining the country’s post-2020 climate actions. • India’s INDC builds on its goal of installing 175 Gigawatts (GW) of renewable power capacity by 2022 by setting a new target to increase the country’s share of non-fossil based installed electric capacity to 40 percent by 2030.

www.insightsonindia.com 85 Insights IAS INSTA 75 Days REVISION PLAN for Prelims 2020 - InstaTests

87. Consider the following statements regarding Bonn Convention 1. Conference of Parties (COP) 13 of the Convention on the Conservation of Migratory Species of Wild Animals (CMS) was organized by India at Chilika lake. 2. The mascot of the event was Gibi, The Great Indian Bustard. 3. The Indian sub-continent is part of the major bird flyway network, i.e, the Central Asian Flyway (CAF). Which of the statement above is/are correct? (a) 1 and 2 only (b) 2 and 3 only (c) 3 only (d) 1, 2 and 3

Solution: B

Convention on the Conservation of Migratory Species of Wild Animals

• The Convention on the Conservation of Migratory Species of Wild Animals, also known as the Convention on Migratory Species or the Bonn Convention. • It is an international agreement that aims to conserve migratory species within their migratory ranges • Conference of Parties (COP) 13 of the Convention on the Conservation of Migratory Species of Wild Animals (CMS) was organized from February 17 to 22 in Gandhinagar, Gujarat. • CMS is an environment treaty under the United Nations Environment Programme (UNEP). • There are 130 parties to the convention and India has been a member since 1983. • India hosted the 13th COP of CMS at Gandhinagar, Gujarat. • Theme – ―Migratory species connect the planet and we welcome them home • Mascot – ―Gibi – The Great Indian Bustard Highlights

• It is the first of a series of international nature-related meetings in 2020 which adopted a new global biodiversity strategy for the next decade – Post-2020 Global Biodiversity Framework • In COP 13, the first ever report on the Status of Migratory Species was presented. • It adopted the Gandhinagar Declaration which calls for migratory species and the concept of ‗ecological connectivity‘ to be integrated and prioritized in the new Framework.

www.insightsonindia.com 86 Insights IAS INSTA 75 Days REVISION PLAN for Prelims 2020 - InstaTests

• CMS Ambassadors Programme was relaunched for terrestrial, avian and aquatic species. • Seven Migratory Species Champions – Germany, India, Italy, Monaco, Norway, the European Commission, and the Environment Agency were acknowledged for their generous contributions to CMS initiatives. • Ethiopia joined the CMS Memorandum of Understanding on the Conservation of Migratory Birds of Prey in Africa and Eurasia (Raptors MOU). • Ethiopia is a strategically important country for the conservation of migratory birds of prey given its location across the East African flyway. • This was the first CMS COP to be inaugurated by a host-country Head of Government. • India has been designated the President of the COP for the next three years (till 2023). • India is home to several migratory species of wildlife, including the snow leopard, Amur falcons, bar- headed geese, black-necked cranes, marine turtles, dugongs and hump-backed whales. • The government of India has been taking necessary actions to protect and conserve migratory marine species. • Seven species that include Dugong, Whale Shark, Marine Turtle (two species), have been identified for preparation of Conservation and Recovery Action Plan. India is temporary home to several migratory animals and birds. The important among these include Amur Falcons, Bar headed Geese, Black necked cranes, Marine turtles, Dugongs, Humpbacked Whales, etc. The Indian sub-continent is also part of the major bird flyway network, i.e, the Central Asian Flyway (CAF) that covers areas between the Arctic and Indian Oceans, and covers at least 279 populations of 182 migratory water bird species, including 29 globally threatened species. India has also launched the National Action Plan for conservation of migratory species under the Central Asian Flyway.

88. Consider the following Statements: 1. The boundaries of a National Park is fixed by state government 2. A Biosphere Reserve is declared to conserve a few specific species of flora and fauna. 3. In a Wildlife Sanctuary, limited biotic interference is permitted. Which of the statements given above is/are correct? (a) 1 only (b) 2 and 3 only (c) 1 and 3 only (d) 1, 2 and 3

www.insightsonindia.com 87 Insights IAS INSTA 75 Days REVISION PLAN for Prelims 2020 - InstaTests

Solution: C

• The state government can fix and alter boundaries of the National Parks with prior consultation and approval with National Board of Wildlife. There is no need to pass an act for alternation of boundaries of National Parks. No human activities are permitted in a National Park. • The major objectives of Biosphere Reserves are: (i) To conserve diversity and integrity of plants, animals and micro-organism; (ii) To promote research on ecological conservation and other environmental aspects and; (iii) To provide facilities for education, awareness and training. • In a Wildlife Sanctuary, limited biotic interference is permitted. Operations such as harvesting of timber, a collection of minor forest products and private ownership rights are permitted so long as they do not interfere with the well-being of animals. In other words, limited biotic interference is allowed. (UPSC 2010)

89. Consider the following statements regarding Tropical Forest Alliance 2020 (TFA 2020) 1. It is a multi-stakeholder partnership platform, initiated to support the implementation of private-sector commitments to remove deforestation from palm oil, beef, soy, and pulp and paper from their supply chains. 2. It is hosted by the World Economic Forum. 3. The Tropical Forest Alliance was founded in 2012 at Rio+20. Which of the statements given above is/are correct? (a) 1 and 2 only (b) 2 and 3 only (c) 1 and 3 only (d) 1, 2 and 3

Solution: D

• The Tropical Forest Alliance (TFA) is a multi-stakeholder partnership platform, initiated to support the implementation of private-sector commitments to remove deforestation from palm oil, beef, soy, and pulp and paper from their supply chains. Hosted by the World Economic Forum, it has grown its partner members throughout the years and continues to bring on board those key actors committed to tackling deforestation. TFA now works with governments, the private sector and civil society actors, indigenous peoples, communities and international organizations to catalyse

www.insightsonindia.com 88 Insights IAS INSTA 75 Days REVISION PLAN for Prelims 2020 - InstaTests

high-impact partnerships to reduce commodity driven deforestation and build a forest positive future. TFA Global and at the Forest Frontier

• The strength of TFA as a platform is to bring together different communities, to identify key forest frontier challenges and solutions. Through bringing experts from around the world our community of purpose turns ideas into action, both at the global and the regional level. It operates regional platforms in Latin America, West and Central Africa, China and Southeast Asia. TFA’s work will also focus on mainstreaming “Forest Positive” landscapes and amplify demand side engagement in major economies such as the US, Europe and in China. Our Donors

• We are grateful for the support of our donors who provide TFA with the necessary resources to support our work. Our funders currently include the Governments of the Netherlands, Norway, Germany, the United Kingdom, and the Gordon and Betty Moore Foundation. • The TFA catalyzes the power of collective action and responsibility by convening, curating and communicating. Our History

• The Tropical Forest Alliance was founded in 2012 at Rio+20 after the Consumer Goods Forum (CGF) committed to zero net deforestation by 2020 for palm oil, soy, beef, and paper and pulp supply chains in 2010. The CGF partnered with the US government to create the public-private alliance with the mission of mobilizing all actors to collaborate in reducing commodity-driven tropical deforestation. Accelerating into the Decade of Delivery

• TFA fosters cross-sector collaboration based on a common and ever-deepening understanding of the barriers and opportunities linked to deforestation-free supply chains. • Its greatest offering is a partnership of champions for deforestation-free global and local economies, making the case for sustainable supply chains as an essential pathway towards a better economy and achievement of the Global Goals.

90. Which of the following regions have been identified as bloom hotspots 1. North Eastern Arabian Sea 2. Coastal waters off Kerala 3. Gulf of Mannar 4. Coastal waters of Gopalpur Select the correct answer using the code given below:

www.insightsonindia.com 89 Insights IAS INSTA 75 Days REVISION PLAN for Prelims 2020 - InstaTests

(a) 1, 2 and 3 only (b) 2, 3 and 4 only (c) 1, 3 and 4 only (d) 1, 2, 3 and 4

Solution: D

Algal Bloom Information Service (ABIS)

• The increasing frequency of algal blooms is a major concern due to its ill effects on fishery, marine life and water quality. INCOIS has developed a service for “Detection and Monitoring of Bloom in the Indian Seas”. • The target users are fishermen, marine fishery resource managers, researchers, ecologists and environmentalists. • The service also complements INCOIS’ marine fishing advisories i.e. Potential Fishing Zone advisories. • INCOIS-ABIS will provide near-real time information on spatio-temporal occurrence and spread of phytoplankton blooms over the North Indian Ocean. • Accordingly, relevant data retrieved from satellites i.e. Sea Surface Temperature , chlorophyll-a, Algal Bloom Index – chlorophyll, rolling chlorophyll anomaly, rolling sea surface temperature anomaly, phytoplankton class/species, phytoplankton size class and a composite image delineating bloom and non-bloom regions will be disseminated daily through ABIS. • In addition, four regions have been identified as bloom hotspots viz. o North Eastern Arabian Sea o coastal waters off Kerala o Gulf of Mannar and o Coastal waters of Gopalpur.

91. Consider the following statements regarding Foreigners Tribunals 1. The foreigners tribunals are quasi-judicial bodies, 2. The powers to constitute foreigners tribunals are vested only with the Centre. Which of the statements given above is/are correct? (a) 1 only (b) 2 only (c) Both 1 and 2 (d) Neither 1 nor 2 www.insightsonindia.com 90 Insights IAS INSTA 75 Days REVISION PLAN for Prelims 2020 - InstaTests

Solution: A

Foreigners Tribunals

• In 1964, the govt brought in the Foreigners (Tribunals) Order. • Composition: Advocates not below the age of 35 years of age with at least 7 years of practice (or) Retired Judicial Officers from the Assam Judicial Service (or) Retired IAS of ACS Officers (not below the rank of Secretary/Addl. Secretary) having experience in quasi-judicial works. Who can setup these tribunals?

• The Ministry of Home Affairs (MHA) has amended the Foreigners (Tribunals) Order, 1964, and has empowered district magistrates in all States and Union Territories to set up tribunals (quasi-judicial bodies) to decide whether a person staying illegally in India is a foreigner or not. • Earlier, the powers to constitute tribunals were vested only with the Centre. • Typically, the tribunals there have seen two kinds of cases: those concerning persons against whom a reference has been made by the border police and those whose names in the electoral roll has a “D”, or “doubtful”, marked against them. Who can approach?

• The amended order (Foreigners (Tribunal) Order, 2019) also empowers individuals to approach the Tribunals. Earlier, only the State administration could move the Tribunal against a suspect. How a person is declared foreigner by these tribunals?

• Foreigners Tribunals, quasi-judicial authorities in Assam, have been deciding on matters pertaining to citizenship in order to identify foreigners. • The process begins by the border police or the Election Commission referring the case of a suspected foreigner to the Foreigners Tribunal. • The tribunal calls on the person to appear before it and prove that they are not a foreigner, and then passes an order in favour or against them.

92. Consider the following statements regarding Nano Mission 1. The International Conference on Nano Science and Nano Technology (ICONSAT) was organized under the aegis of Nano Mission. 2. Nano Mission Council chaired by Prime Minister. 3. It is as an umbrella capacity-building programme. Which of the statements given above is/are correct? (a) 1 and 2 only

www.insightsonindia.com 91 Insights IAS INSTA 75 Days REVISION PLAN for Prelims 2020 - InstaTests

(b) 3 only (c) 1 and 3 only (d) 1, 2 and 3

Solution: C

• The International Conference on Nano Science and Nano Technology (ICONSAT) under the aegis of Nano Mission, Department of Science and Technology (DST) is being held at Kolkata focusing on the recent advances in this frontier research field. Mission on Nano Science and Technology (Nano Mission):

• Launched in 2007. • It is as an “umbrella capacity-building programme”. • The Mission’s programmes will target all scientists, institutions and industry in the country. • It will also strengthen activities in nano science and technology by promoting basic research, human resource development, research infrastructure development, international collaborations, among others. • It will be anchored in the Department of Science and Technology and steered by a Nano Mission Council chaired by an eminent scientist. Outcomes and significance of the mission:

• As a result of the efforts led by the Nano Mission, today, India is amongst the top five nations in the world in terms of scientific publications in nano science and technology (moving from 4th to the 3rd position). • The Nano Mission itself has resulted in about 5000 research papers and about 900 Ph.Ds and also some useful products like nano hydrogel based eye drops, pesticide removal technology for drinking water, water filters for arsenic and fluoride removal, nanosilver based antimicrobial textile coating, etc. • The Nano Mission has thus helped establish a good eco-system in the country to pursue front-ranking basic research and also to seed and nurture application-oriented R&D, focused on useful technologies and products.

93. Consider the following statements regarding Gadgil and Kasturi Rangan committees 1. The Kasturi Rangan report seeks to bring 67% of the Western Ghats under the Ecologically Sensitive Area (ESA) zones. 2. The Gadgil report seeks to bring 37% of the Western Ghats under the Ecologically Sensitive Area (ESA) zones. Which of the statements given above is/are correct? www.insightsonindia.com 92 Insights IAS INSTA 75 Days REVISION PLAN for Prelims 2020 - InstaTests

(a) 1 only (b) 2 only (c) Both 1 and 2 (d) Neither 1 nor 2

Solution: D

• A public interest litigation petition has been filed in the Madras High Court seeking a direction to the Centre and State government to constitute a permanent body for taking serious steps to safeguard the flora, fauna and other natural resources in the Eastern and Western Ghat areas in Tamil Nadu. • The petition is on the basis of the recommendations made by the Madhav Gadgil and Kasturi Rangan committees. What’s the issue? Why there is a need for protection?

• Petitioner contended that the natural resources abundantly available in this area are being properly utilized by other regions, except Tamil Nadu. They are being misutilised and mismanaged not only by the administrators but also by the public at large. • Besides, large-scale plantations of coffee, tea and orchards have been raised in the hills of Western Ghats. Aromatic and valuable trees like sandal are removed illegally. Despite the Wildlife Protection Act, hunting takes place in some pockets. The forests are getting degraded because of illicit collection of firewood, illicit grazing and illicit felling of trees. What did the Gadgil Committee say?

• It defined the boundaries of the Western Ghats for the purposes of ecological management. • It proposed that this entire area be designated as ecologically sensitive area (ESA). • Within this area, smaller regions were to be identified as ecologically sensitive zones (ESZ) I, II or III based on their existing condition and nature of threat. • It proposed to divide the area into about 2,200 grids, of which 75 per cent would fall under ESZ I or II or under already existing protected areas such as wildlife sanctuaries or natural parks. • The committee proposed a Western Ghats Ecology Authority to regulate these activities in the area. Why was Kasturirangan Committee setup?

• None of the six concerned states agreed with the recommendations of the Gadgil Committee, which submitted its report in August 2011.

www.insightsonindia.com 93 Insights IAS INSTA 75 Days REVISION PLAN for Prelims 2020 - InstaTests

• In August 2012, then Environment Minister constituted a High-Level Working Group on Western Ghats under Kasturirangan to “examine” the Gadgil Committee report in a “holistic and multidisciplinary fashion in the light of responses received” from states, central ministries and others. • The Kasturirangan report seeks to bring just 37% of the Western Ghats under the Ecologically Sensitive Area (ESA) zones — down from the 64% suggested by the Gadgil report. Recommendations of Kasturirangan Committee:

• A ban on mining, quarrying and sand mining. • No new thermal power projects, but hydro power projects allowed with restrictions. • A ban on new polluting industries. • Building and construction projects up to 20,000 sq m was to be allowed but townships were to be banned. • Forest diversion could be allowed with extra safeguards.

94. Consider the following statements regarding India Urban Data Exchange (IUDX) 1. It has been launched under smart cities mission 2. It is being implemented by Ministry of Statistics and Programme Implementation 3. It aims to enabling cities to harness full potential of the enormous data being generated in our smart cities. Which of the statements given above is/are correct? (a) 1 only (b) 2 and 3 only (c) 1 and 3 only (d) 1, 2 and 3

Solution: C

India Urban Data Exchange (IUDX)

• IUDX is a research project under smart cities mission being implemented by Union Ministry of Housing and Urban Affairs (MoHUA) in collaboration with Indian Institute of Science (IISc), Bengaluru. • It is an open source software platform aimed at enabling cities to harness full potential of the enormous data being generated in our smart cities • The Ministry of Housing and Urban Affairs (MoHUA) launched an ambitious program for the development of 100 Smart Cities aimed at catalyzing investments in holistic

www.insightsonindia.com 94 Insights IAS INSTA 75 Days REVISION PLAN for Prelims 2020 - InstaTests

infrastructure and services, development of integrated approaches to problem solving, promotion of co-creation and bottom-up innovation using technology, and creation of sustainable outcomes by espousing the circular economy and participatory processes built around communities. • The total investment expected in these cities over a period of 5 years from the date of their selection is expected to be around US$ 30 billion. • Smart Cities, among their many objectives, are evolving as connected, livable, energy efficient, adaptive and resilient cities through deployment of smart solutions.

95. Consider the following pairs of the Ramsar wetlands and the the states they belong to: Ramsar wetlands State 1. Bhoj Wetlands : Madhya Pradesh 2. Chandertal Wetland : Himachal Pradesh 3. Deepor Beel : Assam 4. Surinsar-Mansar Lakes : Jammu and Kashmir Which of the pairs given above is/are correctly matched? (a) 1, 2 and 3 only (b) 2, 3 and 4 only

www.insightsonindia.com 95 Insights IAS INSTA 75 Days REVISION PLAN for Prelims 2020 - InstaTests

(c) 1, 3 and 4 only (d) 1, 2, 3 and 4

Solution: D

RAMSAR WETLANDS SITES (Updated on February, 2019) Sl. No. Name of Site State Location 1 Asthamudi Wetland Kerala 2 Bhitarkanika Mangroves Orissa 3 Bhoj Wetlands Madhya Pradesh 4 Chandertal Wetland Himachal Pradesh 5 Chilka Lake Orissa 6 DeeporBeel Assam 7 East Calcutta Wetlands West Bengal 8 Harike Lake Punjab 9 Hokera Wetland Jammu and Kashmir 10 Kanjli Lake Punjab 11 Keoladeo Ghana NP Rajasthan 12 Kolleru Lake Andhra Pradesh 13 Loktak Lake Manipur 14 Nalsarovar Bird Sanctuary Gujarat 15 Point Calimere Tamil Nadu 16 Pong Dam Lake Himachal Pradesh 17 Renuka Wetland Himachal Pradesh 18 Ropar Lake Punjab 19 Rudrasagar Lake Tripura 20 Sambhar Lake Rajasthan 21 Sasthamkotta Lake Kerala 22 Sunderbans Wetland West Bengal

www.insightsonindia.com 96 Insights IAS INSTA 75 Days REVISION PLAN for Prelims 2020 - InstaTests

23 Surinsar-Mansar Lakes Jammu and Kashmir 24 Tsomoriri Lake Jammu and Kashmir 25 Upper Ganga River (Brijghat to Narora Stretch) Uttar Pradesh 26 VembanadKol Wetland Kerala 27 Wular Lake Jammu & Kashmir

96. Houthis insurgents often seen in the news, are mainly found in (a) Kenya (b) Saudi Arabia (c) Afghanistan (d) Yemen

Solution: D

• The Houthi movement, officially called Ansar Allah and colloquially simply Houthis, is an Islamic political and armed movement that emerged from Sa’dah in northern Yemen in the 1990s. The movement was called Houthis because its founder is from the Houthi tribe.

97. Consider the following statements regarding Census of Marine Life 1. It is a 10-year international effort undertaken in to assess the diversity (how many different kinds), distribution (where they live), and abundance (how many) of marine life. 2. The census is supported by funding from Royal Institution in London. 3. The world’s first comprehensive Census of Marine Life was released in 2020. Which of the statements given above is/are correct? (a) 1 only (b) 2 and 3 only (c) 1 and 3 only (d) 1, 2 and 3

Solution: A

www.insightsonindia.com 97 Insights IAS INSTA 75 Days REVISION PLAN for Prelims 2020 - InstaTests

A DECADE OF DISCOVERY

• 2,700 scientists • 80+ nations • 540 expeditions • US$ 650 million • 2,600+ scientific publications • 6,000+ potential new species • 30 million distribution records and counting These numbers only begin to describe the scope of the Census of Marine Life, a 10-year international effort undertaken in to assess the diversity (how many different kinds), distribution (where they live), and abundance (how many) of marine life—a task never before attempted on this scale. The Census stimulated the discipline of marine science by tackling these issues globally, and engaging some 2,700 scientists from around the globe, who participated in 540 expeditions and countless hours of land-based research. The scientific results were reported on October 4, 2010 at the Royal Institution in London. The world’s first comprehensive Census of Marine Life — past, present, and future — was released in 2010 in London. Initially supported by funding from the Alfred P. Sloan Foundation, the project was successful in generating many times that initial investment in additional support and substantially increased the baselines of knowledge in often underexplored ocean realms, as well as engaging over 2,700 different researchers for the first time in a global collaborative community united in a common goal, and has been described as “one of the largest scientific collaborations ever conducted”.

98. Consider the following statements regarding Sabka Vishwas Scheme 1. It has been launched by Ministry of Social Justice and Empowerment 2. It aims to provide basic amenities to vulnerable sections. Which of the statements given above is/are correct? (a) 1 only (b) 2 only (c) Both 1 and 2 (d) Neither 1 nor 2

Solution: D

www.insightsonindia.com 98 Insights IAS INSTA 75 Days REVISION PLAN for Prelims 2020 - InstaTests

Sabka Vishwas Scheme

• It is a Legacy Dispute Resolution Scheme. • The scheme targets those taxpayers who want to close their pending disputes related to Service Tax and Excise Tax (now subsumed under Goods and Services Tax). Hence, the term ‘legacy’. • This scheme offers amnesty (official pardon) to those who wish to disclose any previously undisclosed tax liability without any penalty or prosecution. • Finance minister Nirmala Sitharaman had unveiled the scheme in the budget for 2019- 20, with an aim to assist taxpayers in clearing the baggage of disputes under legacy taxes (service tax and central excise), which are subsumed in the goods and services tax (GST).

99. Which of the following is/are Natural world heritage sites of India? 1. Great Nicobar National Park 2. Kaziranga National Park 3. Nanda Devi and Valley of Flowers National Parks 4. Sundarbans National Park Select the correct answer using the code given below: (a) 1, 2 and 3 only (b) 2, 3 and 4 only (c) 1, 3 and 4 only (d) 1, 2, 3 and 4

Solution: B

• A UNESCO World Heritage Site is a place that is listed by the United Nations Educational, Scientific and Cultural Organization as of special cultural or physical significance. Natural (7)

• Great Himalayan National Park Conservation Area (2014) • Kaziranga National Park (1985) • Keoladeo National Park (1985) • Manas Wildlife Sanctuary (1985) • Nanda Devi and Valley of Flowers National Parks (1988,2005) • Sundarbans National Park (1987) • Western Ghats (2012)

www.insightsonindia.com 99 Insights IAS INSTA 75 Days REVISION PLAN for Prelims 2020 - InstaTests

Mixed (1)

• Khangchendzonga National Park (2016) http://www.wiienvis.nic.in/Database/whs_pas_8227.aspx

100. Consider the following statements regarding Pradhan Mantri Laghu Vyapari Maan-dhan Yojana: 1. It is a voluntary and contribution based central sector scheme. 2. All small shopkeepers, self-employed persons and retail traders aged between 18- 40 years and with Goods and Service Tax turnover below Rs.1.5 crore can enroll for this scheme. 3. Under this scheme, all beneficiaries eligible for a monthly pension of Rs 3000 after the age of 60. Which of the statements given above is/are correct? (a) 1 and 2 only (b) 2 and 3 only (c) 1 and 3 only (d) 1, 2 and 3

Solution: D

Pradhan Mantri Laghu Vyapari Maan-dhan Yojana:

• It is a voluntary and contribution based central sector scheme. • The government launched the scheme, entailing monthly minimum assured pension of ₹3,000 for the entry age group of 18-40 years after attaining the age of 60 years, with effect from July 22, 2019. • Under the scheme, the government makes matching contribution in the subscribers’ account. • The scheme is based on self-declaration as no documents are required except bank account and Aadhaar Card. Eligibility:

• All small shopkeepers, self-employed persons and retail traders aged between 18-40 years and with Goods and Service Tax (GST) turnover below Rs.1.5 crore can enroll for pension scheme.

www.insightsonindia.com 100 Insights IAS INSTA 75 Days REVISION PLAN for Prelims 2020 - InstaTests

• To be eligible, the applicants should not be covered under the National Pension Scheme, Employees’ State Insurance Scheme and the Employees’ Provident Fund or be an Income Tax assessee.

www.insightsonindia.com 101 Insights IAS INSTA STORIES

www.insightsonindia.com INSIGHTS IAS

www.insightsonindia.com INSIGHTS IAS